Re: Is the "bubble multi-verse" and "qm many-worlds" the same thing?

2018-06-18 Thread Jason Resch
On Mon, Jun 18, 2018 at 2:50 PM,  wrote:

>
>
> On Monday, June 18, 2018 at 1:11:23 PM UTC, Jason wrote:
>>
>>
>>
>> On Mon, Jun 18, 2018 at 7:42 AM,  wrote:
>>
>>>
>>>
>>> On Monday, June 18, 2018 at 12:30:02 PM UTC, Jason wrote:



 On Mon, Jun 18, 2018 at 7:04 AM,  wrote:

>
>>
> *No "but's" about it! That's the 800 pound gorilla in the room that
> Many Worlder's studiously ignore. There's no proposed mechanism to explain
> the copying! They will appeal to the mathematics, which they rely on
> uncritically. But it's obvious that such reliance can lead to ridiculous
> results if taken literally. For example, Maxwell's equations have plane
> wave solutions, but plane waves don't exist! Think about what a plane wave
> is; all peaks and troughs extend to infinity, advancing along an infinite
> plane in every direction. And No, I don't have a classical view of 
> reality,
> but neither do I fall in love with BS. AG*
>
>>
>>
>>
 Rather thank think of it like copying, think of it like this:

 - When a photon hits a semi-silvered mirror, it splits and takes two
 paths.  Now, both the photon's position and its direction are multi-valued.

>>>
>>> *Does it split into two photons, each having the same energy as the
>>> original photon?*
>>>
>>
It's one photon, with a property that is multi-valued.


> * If so, where does the added energy come from.*
>>>
>>
N/A - it's still just one photon


> * Or does it split in half?*
>>>
>>
N/A - it's still just one photon


>
>>> * Or does it take two possible paths, not both paths simultaneously?
>>> TIA, AG*
>>>
>>
If it's velocity property is multi-valued, you could say it follows more
than one path at the same time.
But this is not necessary the case. The superposition might be in terms of
polarization, rather than its velocity, and in that case it's only takes
one path.

Jason


>
>> Think of it being only a change to the photon's properties.
>>
>
> *If you are able, and have the time, please answer my question above,
> specifically. TIA, AG*
>
>
>> It's not really two photons, it's a single photon, now holding multiple
>> values for some of its properties (which include velocity, position,
>> frequency, polarization, etc.)  After interacting with a semi-silvered
>> mirror, the same thing always happens to a photon, it takes on multiple
>> values for its velocity.
>>
>> Jason
>>
>>
>>
>>>
>>>
 - We can use full-silvered mirrors to redirect this "multi-valued"
 photon such that both of its positions will converge (while nonetheless
 traveling in different directions on that intercept course)
 - We can place an electron at that point of interception, such that
 this multi-velocity photon interacts with and strikes this electron (which
 right now only has one position and zero velocity)
 - When the multi-velocity photon hits the electron, the electron now
 has two velocities (you could view it as in one case, it was hit from
 below, and in another case it was hit from the side).

 So we see a clear case, where a photon, being in a super-position of
 states, when it interacts with an electron, it's multiple states transfer
 to that electron such that it now ends up in a super position, and will
 have different positions and velocities over time.  Anything that interacts
 with that electron, or doesn't interact with it will also become
 multi-valued.

 Let's say there is a detection screen to the right of the electron,
  the multi-valued electron interacts with the atoms in the detection
 screen. Because the electron's position is multi-valued, and only one of
 the electron's velocity's intercept the screen, the atoms in the detection
 screen also enter a super position, of having interacted with the electron
 and having not interacted with the electron (the upward moving electron
 velocity misses the screen).  So now there is a superposition of states in
 the room where a flash flash of light from the screen has occurred, and
 where no flash of light on the screen has occurred.

 You can continue this example forward until you end up with yourself in
 the room with two different brains, one where it is encoding memories of
 having seen a flash of light and another where it did not.  Nothing special
 is itnroduced by the observer, the observer is, afterall, just a collection
 of particles, each of which may or may not enter a superposition of states,
 depending on whether or not they interact with any particle that is in a
 super position.

 This is how the "copying" works.  It's just state transfer.  Think of
 it as:

 x is in a superposition of +3 and - 3.  In other words x = ±3
 y is 5

 y interacts with x as follows: y = y * x

 What this means is y = 5 * (±3)

 Now y is multi-valued, in 

Re: Is the "bubble multi-verse" and "qm many-worlds" the same thing?

2018-06-18 Thread agrayson2000


On Monday, June 18, 2018 at 1:11:23 PM UTC, Jason wrote:
>
>
>
> On Mon, Jun 18, 2018 at 7:42 AM, > 
> wrote:
>
>>
>>
>> On Monday, June 18, 2018 at 12:30:02 PM UTC, Jason wrote:
>>>
>>>
>>>
>>> On Mon, Jun 18, 2018 at 7:04 AM,  wrote:
>>>

>
 *No "but's" about it! That's the 800 pound gorilla in the room that 
 Many Worlder's studiously ignore. There's no proposed mechanism to explain 
 the copying! They will appeal to the mathematics, which they rely on 
 uncritically. But it's obvious that such reliance can lead to ridiculous 
 results if taken literally. For example, Maxwell's equations have plane 
 wave solutions, but plane waves don't exist! Think about what a plane wave 
 is; all peaks and troughs extend to infinity, advancing along an infinite 
 plane in every direction. And No, I don't have a classical view of 
 reality, 
 but neither do I fall in love with BS. AG*

>
>
>
>>> Rather thank think of it like copying, think of it like this:
>>>
>>> - When a photon hits a semi-silvered mirror, it splits and takes two 
>>> paths.  Now, both the photon's position and its direction are multi-valued.
>>>
>>
>>
>> *Does it split into two photons, each having the same energy as the 
>> original photon? If so, where does the added energy come from. Or does it 
>> split in half? Or does it take two possible paths, not both paths 
>> simultaneously? TIA, AG*
>>
>
> Think of it being only a change to the photon's properties. 
>

*If you are able, and have the time, please answer my question above, 
specifically. TIA, AG*
 

> It's not really two photons, it's a single photon, now holding multiple 
> values for some of its properties (which include velocity, position, 
> frequency, polarization, etc.)  After interacting with a semi-silvered 
> mirror, the same thing always happens to a photon, it takes on multiple 
> values for its velocity.
>
> Jason
>
>  
>
>>  
>>
>>> - We can use full-silvered mirrors to redirect this "multi-valued" 
>>> photon such that both of its positions will converge (while nonetheless 
>>> traveling in different directions on that intercept course)
>>> - We can place an electron at that point of interception, such that this 
>>> multi-velocity photon interacts with and strikes this electron (which right 
>>> now only has one position and zero velocity)
>>> - When the multi-velocity photon hits the electron, the electron now has 
>>> two velocities (you could view it as in one case, it was hit from below, 
>>> and in another case it was hit from the side).
>>>
>>> So we see a clear case, where a photon, being in a super-position of 
>>> states, when it interacts with an electron, it's multiple states transfer 
>>> to that electron such that it now ends up in a super position, and will 
>>> have different positions and velocities over time.  Anything that interacts 
>>> with that electron, or doesn't interact with it will also become 
>>> multi-valued.
>>>
>>> Let's say there is a detection screen to the right of the electron,  the 
>>> multi-valued electron interacts with the atoms in the detection screen. 
>>> Because the electron's position is multi-valued, and only one of the 
>>> electron's velocity's intercept the screen, the atoms in the detection 
>>> screen also enter a super position, of having interacted with the electron 
>>> and having not interacted with the electron (the upward moving electron 
>>> velocity misses the screen).  So now there is a superposition of states in 
>>> the room where a flash flash of light from the screen has occurred, and 
>>> where no flash of light on the screen has occurred.
>>>
>>> You can continue this example forward until you end up with yourself in 
>>> the room with two different brains, one where it is encoding memories of 
>>> having seen a flash of light and another where it did not.  Nothing special 
>>> is itnroduced by the observer, the observer is, afterall, just a collection 
>>> of particles, each of which may or may not enter a superposition of states, 
>>> depending on whether or not they interact with any particle that is in a 
>>> super position.
>>>
>>> This is how the "copying" works.  It's just state transfer.  Think of it 
>>> as:
>>>
>>> x is in a superposition of +3 and - 3.  In other words x = ±3
>>> y is 5
>>>
>>> y interacts with x as follows: y = y * x
>>>
>>> What this means is y = 5 * (±3)
>>>
>>> Now y is multi-valued, in a superposition of 15 and -15.
>>>
>>> No magic full scale copying of entire universes.  It's just particle 
>>> states can be multi-valued, and anything that interacts with a multi-valued 
>>> particle is effected by it being multi-valued.
>>>
>>> Jason
>>>
>> -- 
>> You received this message because you are subscribed to the Google Groups 
>> "Everything List" group.
>> To unsubscribe from this group and stop receiving emails from it, send an 
>> email to everything-li...@googlegroups.com .
>> To post to this group, send email 

Re: Is the "bubble multi-verse" and "qm many-worlds" the same thing?

2018-06-18 Thread agrayson2000


On Monday, June 18, 2018 at 8:33:23 PM UTC, Jason wrote:
>
>
>
> On Mon, Jun 18, 2018 at 2:50 PM, > 
> wrote:
>
>>
>>
>> On Monday, June 18, 2018 at 1:11:23 PM UTC, Jason wrote:
>>>
>>>
>>>
>>> On Mon, Jun 18, 2018 at 7:42 AM,  wrote:
>>>


 On Monday, June 18, 2018 at 12:30:02 PM UTC, Jason wrote:
>
>
>
> On Mon, Jun 18, 2018 at 7:04 AM,  wrote:
>
>>
>>>
>> *No "but's" about it! That's the 800 pound gorilla in the room that 
>> Many Worlder's studiously ignore. There's no proposed mechanism to 
>> explain 
>> the copying! They will appeal to the mathematics, which they rely on 
>> uncritically. But it's obvious that such reliance can lead to ridiculous 
>> results if taken literally. For example, Maxwell's equations have plane 
>> wave solutions, but plane waves don't exist! Think about what a plane 
>> wave 
>> is; all peaks and troughs extend to infinity, advancing along an 
>> infinite 
>> plane in every direction. And No, I don't have a classical view of 
>> reality, 
>> but neither do I fall in love with BS. AG*
>>
>>>
>>>
>>>
> Rather thank think of it like copying, think of it like this:
>
> - When a photon hits a semi-silvered mirror, it splits and takes two 
> paths.  Now, both the photon's position and its direction are 
> multi-valued.
>

 *Does it split into two photons, each having the same energy as the 
 original photon?*

>>>
> It's one photon, with a property that is multi-valued.
>  
>
>> * If so, where does the added energy come from.*

>>>
> N/A - it's still just one photon
>  
>
>> * Or does it split in half?*

>>>
> N/A - it's still just one photon
>  
>
>>
 * Or does it take two possible paths, not both paths simultaneously? 
 TIA, AG*

>>>
> If it's velocity property is multi-valued, you could say it follows more 
> than one path at the same time.
> But this is not necessary the case. The superposition might be in terms of 
> polarization, rather than its velocity, and in that case it's only takes 
> one path.
>  
> Jason
>

*So it takes one of two paths, presumably with a 50% probability. How does 
this result in copies, of the photon or possibly entire universes?  Sounds 
like you're grasping at straws to explain the copying mechanism. AG*

>  
>
>>
>>> Think of it being only a change to the photon's properties. 
>>>
>>
>> *If you are able, and have the time, please answer my question above, 
>> specifically. TIA, AG*
>>  
>>
>>> It's not really two photons, it's a single photon, now holding multiple 
>>> values for some of its properties (which include velocity, position, 
>>> frequency, polarization, etc.)  After interacting with a semi-silvered 
>>> mirror, the same thing always happens to a photon, it takes on multiple 
>>> values for its velocity.
>>>
>>> Jason
>>>
>>>  
>>>
  

> - We can use full-silvered mirrors to redirect this "multi-valued" 
> photon such that both of its positions will converge (while nonetheless 
> traveling in different directions on that intercept course)
> - We can place an electron at that point of interception, such that 
> this multi-velocity photon interacts with and strikes this electron 
> (which 
> right now only has one position and zero velocity)
> - When the multi-velocity photon hits the electron, the electron now 
> has two velocities (you could view it as in one case, it was hit from 
> below, and in another case it was hit from the side).
>
> So we see a clear case, where a photon, being in a super-position of 
> states, when it interacts with an electron, it's multiple states transfer 
> to that electron such that it now ends up in a super position, and will 
> have different positions and velocities over time.  Anything that 
> interacts 
> with that electron, or doesn't interact with it will also become 
> multi-valued.
>
> Let's say there is a detection screen to the right of the electron, 
>  the multi-valued electron interacts with the atoms in the detection 
> screen. Because the electron's position is multi-valued, and only one of 
> the electron's velocity's intercept the screen, the atoms in the 
> detection 
> screen also enter a super position, of having interacted with the 
> electron 
> and having not interacted with the electron (the upward moving electron 
> velocity misses the screen).  So now there is a superposition of states 
> in 
> the room where a flash flash of light from the screen has occurred, and 
> where no flash of light on the screen has occurred.
>
> You can continue this example forward until you end up with yourself 
> in the room with two different brains, one where it is encoding memories 
> of 
> having seen a flash of light and another where it did not.  Nothing 
> special 

Re: Is the Continuum Hypothesis a) really true or really false, or b) something else ?

2018-06-18 Thread Bruno Marchal

> On 11 Jun 2018, at 18:08, John Clark  wrote:
> 
> On Mon, Jun 11, 2018 at 5:16 AM, Bruno Marchal  > wrote:
> 
> ​> ​why in the world did you say " With mechanism, obviously a soul, or a 
> first person experience can be duplicated from a third person pov. But not 
> from a first person pov”?
> 
> ​> ​I think this has ben explained many times,
> 
> I think the internet must have failed many times because it failed to deliver 
> any of those explanations to me.
> 
> > With mechanism we can duplicate you, in W and M, say. For an external 
> > observer who accept mechanism, there is a you conscious in W and there is a 
> > you conscious in M. In that sense (the 3-1 sense) your soul has been 
> > duplicated relatively to the external observer.
> 
> OK, I have no problem with any of that.
> 
> > But let us ask both the you in M and the you in W: both confirms that from 
> > their point of view, they have not felt any duplication, [...] at no moment 
> > do they have a FIRST PERSON experience of a split.
> 
> Exactly! If the copy had noticed the duplication that would mean the 
> duplication process was imperfect and caused a large enough difference 
> between the copy and the original that the copy noticed a discontinuous 
> change the instant the copy button was pressed but the original noticed no 
> change; however this is NOT what happened, if there was any imperfections in 
> the copy process at all the change was too small for the copy to notice that 
> anything unusual happened when the copy button was pressed. 
> 
> > and the other copy is no more attached to their personal experience. It is 
> > a doppelgänger. They might feel intimate with their   dippelganger in some 
> > intellectual way, but without magic or telepathy, despite they re both the 
> > “H-guy”, they have become independent person,
> 
> Forget telepathy! If 2 identical grandfather clocks are running properly and 
> set to the same time and you come back an hour later and notice they still 
> show the same time you don’t need to invoke telepathy to explain it, indeed 
> if they DIDN’T show the same time then you’d need to resort to some new 
> spooky action at a distance effect previously unknown to science.
> 
> > Their soul has been maintained private and integral: no soul duplication in 
> > the soul’s first personal view.
> 
> Stating something is not the same as proving something. You start with the 
> axiom that the “soul” can’t be duplicated, and end your “proof” by claiming 
> you’ve proven it.


I start from the fact that any exact prediction is refuted in both copies’ 
diary/memory. 





> 
> > See above
> 
> Why?
> 
> >>There is nothing indeterminate about that, its all 100% predictable.
> 
> >Ok, what is your algorithm in Helsinki?
> 
> Seeing Moscow will turn the Helsinki man into the Moscow man and seeing 
> Washington will turn the Helsinki man into the Washington man with 100% 
> certainty and no indeterminacy whatsoever.

That fails to give the prediction made in Helsinki about the immediate future 
experience. The H-guy know that he will survive in both place, but also but 
that in both places he will felt like surviving in only one place from its 
first person point of view.
That is the reason why it cannot predict which one will be lived in particular, 

Again, you fail to address the first experience lived by the two copies.

Bruno


> 
> > how the H-person, when still in Helsinki could predict who he will feel to 
> > be?
> 
> A prediction can’t be made until it is clear exactly who Mr. He is. Forget 
> people duplicating machines, if Mr. He means the man experiencing Helsinki on 
> June 11 2018 at 14:36:09 Coordinated Universal Time then Mr. He will 
> experience no city and no nothing tomorrow because by definition Mr. He will 
> not exist then. However if Mr. He means the person who remembers experiencing 
> Helsinki on June 11 2018 at 14:36:09 Coordinated Universal Time and if Mr. He 
> is duplicated then the fact that there are 2 answers to your question is no 
> more metaphysical or indeterminate or profound than the fact that there and 2 
> correct answers to the question “What is the value of X in this quadratic 
> equation X^2=4 ?”. Don’t you think it would be silly to demand to know the 
> one and only one true answer?
> 
> > You ignore the work of Theaetetus,
> 
> Theaetetus wasn’t a person, Theaetetus was one of Plato’s dialogs. And the 
> time reading Plato is time spent not reading FAR more important things.
> 
> > and apparently even Diophantus, who founded Algebra
> 
> If this list existed one thousand eight hundred years ago I’d be talking a 
> lot about Diophantus too, but there have been a few interesting developments 
> since the days of Diophantus, such as the far more recent discovery made in 
> 1530 on how to solve cubic equations, something Diophantus had no idea how to 
> do.
> 
> >>why would anybody working on modern scientific problems be interested in 
> 

Re: Is the "bubble multi-verse" and "qm many-worlds" the same thing?

2018-06-18 Thread Jason Resch
On Mon, Jun 18, 2018 at 12:01 AM, Brent Meeker  wrote:

>
>
> On 6/17/2018 2:24 PM, Jason Resch wrote:
>
>
>
> On Sunday, June 17, 2018,  wrote:
>
>>
>>
>> On Sunday, June 17, 2018 at 12:29:35 PM UTC, Jason wrote:
>>>
>>>
>>>
>>> On Sun, Jun 17, 2018 at 6:26 AM,  wrote:
>>>


 On Sunday, June 17, 2018 at 10:15:05 AM UTC, Jason wrote:
>
>
>
> On Sun, Jun 17, 2018 at 12:12 AM,  wrote:
>
>>
>>
>> * why do you prefer the MWI compared to the Transactional
>> Interpretation? I see both as absurd. so I prefer to assume the wf is 
>> just
>> epistemic, and/or that we have some holes in the CI which have yet to be
>> resolved. AG *
>> --
>>
>
>
> 1. It's the simplest theory: "MWI" is just the Schrodinger equation,
> nothing else. (it doesn't say Schrodinger's equation only applies
> sometimes, or only at certain scales)
>
> 2. It explains more while assuming less (it explains the appearance of
> collapse, without having to assume it, thus is preferred by Occam's razor)
>
> 3. Like every other successful physical theory, it is linear,
> reversible (time-symmetric), continuous, deterministic and does not 
> require
> faster than light influences nor retrocausalities
>
> 4. Unlike single-universe or epistemic interpretations, "WF is real"
> with MWI is the only way we know how to explain the functioning of quantum
> computers (now up to 51 qubits)
>
> 5. Unlike copenhagen-type theories, it attributes no special physical
> abilities to observers or measurement devices
>
> 6. Most of all, theories of everything that assume a reality
> containing all possible observers and observations lead directly to
> laws/postulates of quantum mechanics (see Russell Standish's Theory
> of Nothing ,
> Chapter 7 and Appendix D).
>
> Given #6, we should revise our view. It is not MWI and QM that should
> convince us of many worlds, but rather the assumption of many worlds (an
> infinite and infinitely varied reality) that gives us, and *explains *all
> the weirdness of QM. This should overwhelmingly convince us of MWI-type
> everything theories over any single-universe interpretation of quantum
> mechanics, which is not only absurd, but completely devoid of explanation.
> With the assumption of a large reality, QM is made explainable and
> understandable: as a theory of observation within an infinite reality.
>
> Jason
>

 * You forgot #7. It asserts multiple, even infinite copies of an
 observer, replete with memories, are created when an observer does a simple
 quantum experiment. So IMO the alleged "cure" is immensely worse than the
 disease, CI, that is, just plain idiotic. AG *


>>> There are many atoms, many planets, many solar systems, many galaxies,
>>> many Hubble volumes, and it is believed many universes.  On what basis are
>>> you so certain there aren't many histories? (That is, other states in the
>>> wave function that are predicted to be there by our well established
>>> scientific theories, but which the theory explains we cannot see or
>>> interact with except in very limited controlled manners)?
>>>
>>> If you find MWI distasteful you might prefer to think of it as the
>>> many-minds interpretation as described by Heinz-Dieter Zeh, or the
>>> "zero-universe interpretation" as explained by Ron Garrett:
>>> https://www.youtube.com/watch?v=dEaecUuEqfc
>>>
>>> I think you are hung up on the "creation", I think it is conceptually
>>> easier to grasp under the understanding that it is all already there.  If
>>> you look at the homepage of Wei Dai (who founded this e-mailing list
>>>  20 years ago) he outlines what
>>> he calls "a very simple interpretation of quantum mechanics
>>> " which is basically this:
>>> all the states are already there.
>>>
>>
>> *Sounds like Super-Determinism proposed by t'Hooft, and referenced
>> yesterday by Brent, which proposes the universe knows beforehand what kind
>> of experiment Joe the Plumber will perform. Too ridiculous for my tastes,
>> and of course untestable. IMO, one of the "achievements" of quantum theory
>> is to make otherwise intelligent persons totally gullible in what they
>> believe as plausible.  AG*
>>
>>>
>>>
> I agree with you about super derterminism being too ridiculous to believe.
> But super derterminism is a different animal from "block time".  Super
> derterminism is the idea that the universe conspires against all
> experimenters and knows what they will measure before they measure it, and
> chooses values they will measure to make things work out.  It's reminiscent
> of Descartes evil demon. It requires an evil God.
>
>
> You've anthropomorphized the universe.  The universe doesn't 

Re: Is the "bubble multi-verse" and "qm many-worlds" the same thing?

2018-06-18 Thread Bruno Marchal

> On 14 Jun 2018, at 05:30, Brent Meeker  wrote:
> 
> 
> 
> On 6/13/2018 4:30 PM, Jason Resch wrote:
>> 
>> Physical Theories, Eternal Inflation, and Quantum Universe 
>> , Yasunori Nomura
>> 
>> We conclude that the eternally inflating multiverse and many worlds in
>> quantum mechanics are the same. Other important implications include: global 
>> spacetime
>> can be viewed as a derived concept; the multiverse is a transient phenomenon 
>> during the
>> world relaxing into a supersymmetric Minkowski state. We also present a 
>> theory of “initial
>> conditions” for the multiverse. By extrapolating our framework to the 
>> extreme, we arrive at a
>> picture that the entire multiverse is a fluctuation in the stationary, 
>> fractal “mega-multiverse,”
>> in which an infinite sequence of multiverse productions occurs.
>> 
>> "Therefore, we conclude that the multiverse is the same as (or a specific 
>> manifestation
>> of ) many worlds in quantum mechanics.”


I tend to agree with this. I have used multiverse, like Deutsch, always in the 
sense of Everett Many-Worlds, with some nuances like “‘many consistent relative 
 histories” à la Griffith and Omnes. But with quantum cosmology, the 
many-worlds can lead to “many universes” in a more concrete sense, which should 
not be confused with the interpretation of quantum mechanics itself, but of 
general relativity when quantised in a way or in another.



> 
> That makes the multiple universes hidden variables. So they must interact 
> (non-locally) else they are rule out by Bell theorem violations.

Maybe, if the nuances between cosmology and the quantum are not addressed. 
Normally the Everett “many-worlds” do not interact at all (that would be non 
local indeed), but interfere statically, like mechanism predicts in arithmetic.

Bruno




> 
> Brent
> 
>> 
>> "In eternal inflation, however, one first picks a causal patch; then one 
>> looks for observers in it.” Our framework does not follow this approach. We 
>> instead pick an observer first, and then construct the relevant spacetime 
>> regions associated with it.
>> 
>> Instead of admitting the existence of the “beginning,” we may require that 
>> the quantum observer principle is respected for the whole history of 
>> spacetime. In this case, the beginning of our multiverse is a fluctuation of 
>> a larger structure, whose beginning is also a fluctuation of an even larger 
>> structure, and this series goes on forever. This leads to the picture that 
>> our multiverse arises as a fluctuation in a huge, stationary 
>> “megamultiverse,” which has a fractal structure."
>> 
>> The Multiverse Interpretation of Quantum Mechanics 
>> , Raphael Bousso and Leonard Susskind
>> 
>> In both the many-worlds interpretation of quantum mechanics and the 
>> multiverse
>> of eternal inflation the world is viewed as an unbounded collection of 
>> parallel universes.
>> A view that has been expressed in the past by both of us is that there is no 
>> need to
>> add an additional layer of parallelism to the multiverse in order to 
>> interpret quantum
>> mechanics. To put it succinctly, the many-worlds and the multiverse are the 
>> same
>> thing [1].
>> 
>> Jason
>> -- 
>> You received this message because you are subscribed to the Google Groups 
>> "Everything List" group.
>> To unsubscribe from this group and stop receiving emails from it, send an 
>> email to everything-list+unsubscr...@googlegroups.com 
>> .
>> To post to this group, send email to everything-list@googlegroups.com 
>> .
>> Visit this group at https://groups.google.com/group/everything-list 
>> .
>> For more options, visit https://groups.google.com/d/optout 
>> .
> 
> 
> -- 
> You received this message because you are subscribed to the Google Groups 
> "Everything List" group.
> To unsubscribe from this group and stop receiving emails from it, send an 
> email to everything-list+unsubscr...@googlegroups.com 
> .
> To post to this group, send email to everything-list@googlegroups.com 
> .
> Visit this group at https://groups.google.com/group/everything-list 
> .
> For more options, visit https://groups.google.com/d/optout 
> .

-- 
You received this message because you are subscribed to the Google Groups 
"Everything List" group.
To unsubscribe from this group and stop receiving emails from it, send an email 
to everything-list+unsubscr...@googlegroups.com.
To post to this group, send email to everything-list@googlegroups.com.
Visit this group at https://groups.google.com/group/everything-list.
For more options, visit 

Re: Is the "bubble multi-verse" and "qm many-worlds" the same thing?

2018-06-18 Thread Jason Resch
On Sun, Jun 17, 2018 at 6:24 PM, Bruce Kellett 
wrote:

> From: Jason Resch < jasonre...@gmail.com>
>
>
> On Sun, Jun 17, 2018 at 6:42 AM, Bruce Kellett <
> bhkell...@optusnet.com.au> wrote:
>
>> From: Jason Resch 
>>
>> On Sun, Jun 17, 2018 at 12:12 AM, < 
>> agrayson2...@gmail.com> wrote:
>>
>>>
>>>
>>> * why do you prefer the MWI compared to the Transactional
>>> Interpretation? I see both as absurd. so I prefer to assume the wf is just
>>> epistemic, and/or that we have some holes in the CI which have yet to be
>>> resolved. AG *
>>>
>>
>>
>> 1. It's the simplest theory: "MWI" is just the Schrodinger equation,
>> nothing else. (it doesn't say Schrodinger's equation only applies
>> sometimes, or only at certain scales)
>>
>>
>> Well no, it is an interpretation of the SE, involving the reification of
>> the wave function. So it is not 'just' the Schrödinger equation.
>>
>
> It is a theory, in that it is fully mathematical and makes predictions.
> Other so called interpretations CI etc. are not mathematical theories
> because they don't say when or why or under what circumstances
> Schrodinger's equation stops working.  As Max Tegmark said:
>
> “I disagree that the distinction between Everett and Copenhagen is ‘just
> interpretation’. The former is a mathematical theory, the latter is not.
> The former says simply that the Schrödinger equation always applies. The
> latter says that it only applies sometimes, but doesn't given an equation
> specifying when it doesn't apply (when the so-called collapse is supposed
> to happen). If someone were to come up with such an equation, then the two
> theories would be mathematically different and you might hope to make an
> experiment to test which one is right.”
>
>
> Decoherence theory effectively answers such objections.
>

Yes and if you accept Decoherence as an explanation for the appearance of
wave function collapse, then you should elide the "wave function collapse"
postulate from your theory (as it has already been explained from the other
existing postulates). And what is QM without the wave function collapse
postulate?  It's just the Schrodinger equation, applying at all times and
all scales.


>
>
>
> You speak of reification of the wave function as if it is something
> special.  In what other physical theory is something postulated one theory,
> and a different theory is when that same thing is postulated, but is also
> "really real"?  Is the theory of quarks distinct from another theory of
> quarks that holds them to be really real?
>
> David Deutsch comments on the absurdity of this:
>
> “Schrödinger also”, David Deutsch notes, “had the basic idea of parallel
> universes shortly before Everett, but he didn't publish it. He mentioned it
> in a lecture in Dublin, in which he predicted that the audience would think
> he was crazy. Isn't that a strange assertion coming from a Nobel Prize
> winner—that he feared being considered crazy for claiming that his
> equation, the one that he won the Nobel Prize for, might be true.”
>
>
> Schrödinger was also the originator of the idea of a collapse of the wave
> function. He saw that his wave function necessitated a collapse of his wave
> to a point particle interaction in the majority of measurements.
>

When he, and all other physicists were still under the spell of believing
there must only be one universe that remains real. He and other physicists
at the time believed otherwise it would lead to a "jellificiation" of
states.  It was only later in life he began to appreciate the full
implications of his theory. And Decoherence in 1952, provided a means to
avoid that jellification of states.

See: https://goo.gl/49xwzJ


>
>
>
> 2. It explains more while assuming less (it explains the appearance of
>> collapse, without having to assume it, thus is preferred by Occam's razor)
>>
>>
>> Maybe the collapse is real.
>>
>
>
> But to assume this is like assuming there are invisible and undetectable
> "motive demons" operating within a car engine that are necessary to make
> the car engine work, when we have another perfectly valid way of explaining
> everything the car engine does without having to assume these motive
> demons.  I don't see the point when we have a theory that explains all the
> facts before us.
>
>
> Maybe it just means that we don't yet fully understand the collapse. There
> are plenty of possibilities that don't resort to magic.
>
>
I agree.

But what facts about our observations of collapse are not already fully
explained by Decoherence?

In other words, what is left to solve about it? I thought Decoherence
solved this (back in 1952 with Bohm).


>
> 3. Like every other successful physical theory, it is linear, reversible
>> (time-symmetric), continuous, deterministic and does not require faster
>> than light influences nor retrocausalities
>>
>>
>> MWI is still a non-local theory. FTL influences or not, QM is
>> intrinsically non-local.
>>
>
>
> When you say non-local what type of non-locality do 

Re: Is the "bubble multi-verse" and "qm many-worlds" the same thing?

2018-06-18 Thread Jason Resch
On Sun, Jun 17, 2018 at 9:57 PM, Brent Meeker  wrote:

>
>
> On 6/17/2018 4:43 AM, Telmo Menezes wrote:
>
>> On 17 June 2018 at 13:26,   wrote:
>>
>>>
>>> On Sunday, June 17, 2018 at 10:15:05 AM UTC, Jason wrote:
>>>


 On Sun, Jun 17, 2018 at 12:12 AM,  wrote:

>
>
>   why do you prefer the MWI compared to the Transactional
> Interpretation?
> I see both as absurd. so I prefer to assume the wf is just epistemic,
> and/or
> that we have some holes in the CI which have yet to be resolved. AG
>
> --
>


 1. It's the simplest theory: "MWI" is just the Schrodinger equation,
 nothing else. (it doesn't say Schrodinger's equation only applies
 sometimes,
 or only at certain scales)

 2. It explains more while assuming less (it explains the appearance of
 collapse, without having to assume it, thus is preferred by Occam's
 razor)

 3. Like every other successful physical theory, it is linear, reversible
 (time-symmetric), continuous, deterministic and does not require faster
 than
 light influences nor retrocausalities

 4. Unlike single-universe or epistemic interpretations, "WF is real"
 with
 MWI is the only way we know how to explain the functioning of quantum
 computers (now up to 51 qubits)

 5. Unlike copenhagen-type theories, it attributes no special physical
 abilities to observers or measurement devices

 6. Most of all, theories of everything that assume a reality containing
 all possible observers and observations lead directly to
 laws/postulates of
 quantum mechanics (see Russell Standish's Theory of Nothing, Chapter 7
 and
 Appendix D).

 Given #6, we should revise our view. It is not MWI and QM that should
 convince us of many worlds, but rather the assumption of many worlds (an
 infinite and infinitely varied reality) that gives us, and explains all
 the
 weirdness of QM. This should overwhelmingly convince us of MWI-type
 everything theories over any single-universe interpretation of quantum
 mechanics, which is not only absurd, but completely devoid of
 explanation.
 With the assumption of a large reality, QM is made explainable and
 understandable: as a theory of observation within an infinite reality.

 Jason

>>>
>>> You forgot #7. It asserts multiple, even infinite copies of an observer,
>>> replete with memories, are created when an observer does a simple quantum
>>> experiment. So IMO the alleged "cure" is immensely worse than the
>>> disease,
>>> CI, that is, just plain idiotic. AG
>>>
>> It is important to make the distinction between our intuition and
>> common sense and actual formal reasoning. The former can guide the
>> latter very successfully, but the history of science teaches us that
>> this is not always the case. You don't provide an argument, you just
>> present your gut feeling as if it were the same thing as irrefutable
>> fact.
>>
>
> I think Scott Aaronson has the right attitude toward this:
>
> https://www.scottaaronson.com/blog/?p=326
>
>
As such a strong believer in quantum computers (he's staked $100,000 of his
own money on the future construction of large scale quantum computers), I
would love to ask Scott Aaronson what he thinks about running a conscious
AI on such a quantum computer.  That trivially leads to "many worlds" at
least as seen by that AI.

QM also tells us that Wigner's friend, is no different from that "AI
running on a quantum computer".

Jason

-- 
You received this message because you are subscribed to the Google Groups 
"Everything List" group.
To unsubscribe from this group and stop receiving emails from it, send an email 
to everything-list+unsubscr...@googlegroups.com.
To post to this group, send email to everything-list@googlegroups.com.
Visit this group at https://groups.google.com/group/everything-list.
For more options, visit https://groups.google.com/d/optout.


Re: Is the "bubble multi-verse" and "qm many-worlds" the same thing?

2018-06-18 Thread Telmo Menezes
On 18 June 2018 at 04:57, Brent Meeker  wrote:
>
>
> On 6/17/2018 4:43 AM, Telmo Menezes wrote:
>>
>> On 17 June 2018 at 13:26,   wrote:
>>>
>>>
>>> On Sunday, June 17, 2018 at 10:15:05 AM UTC, Jason wrote:



 On Sun, Jun 17, 2018 at 12:12 AM,  wrote:
>
>
>
>   why do you prefer the MWI compared to the Transactional
> Interpretation?
> I see both as absurd. so I prefer to assume the wf is just epistemic,
> and/or
> that we have some holes in the CI which have yet to be resolved. AG
>
> --



 1. It's the simplest theory: "MWI" is just the Schrodinger equation,
 nothing else. (it doesn't say Schrodinger's equation only applies
 sometimes,
 or only at certain scales)

 2. It explains more while assuming less (it explains the appearance of
 collapse, without having to assume it, thus is preferred by Occam's
 razor)

 3. Like every other successful physical theory, it is linear, reversible
 (time-symmetric), continuous, deterministic and does not require faster
 than
 light influences nor retrocausalities

 4. Unlike single-universe or epistemic interpretations, "WF is real"
 with
 MWI is the only way we know how to explain the functioning of quantum
 computers (now up to 51 qubits)

 5. Unlike copenhagen-type theories, it attributes no special physical
 abilities to observers or measurement devices

 6. Most of all, theories of everything that assume a reality containing
 all possible observers and observations lead directly to laws/postulates
 of
 quantum mechanics (see Russell Standish's Theory of Nothing, Chapter 7
 and
 Appendix D).

 Given #6, we should revise our view. It is not MWI and QM that should
 convince us of many worlds, but rather the assumption of many worlds (an
 infinite and infinitely varied reality) that gives us, and explains all
 the
 weirdness of QM. This should overwhelmingly convince us of MWI-type
 everything theories over any single-universe interpretation of quantum
 mechanics, which is not only absurd, but completely devoid of
 explanation.
 With the assumption of a large reality, QM is made explainable and
 understandable: as a theory of observation within an infinite reality.

 Jason
>>>
>>>
>>> You forgot #7. It asserts multiple, even infinite copies of an observer,
>>> replete with memories, are created when an observer does a simple quantum
>>> experiment. So IMO the alleged "cure" is immensely worse than the
>>> disease,
>>> CI, that is, just plain idiotic. AG
>>
>> It is important to make the distinction between our intuition and
>> common sense and actual formal reasoning. The former can guide the
>> latter very successfully, but the history of science teaches us that
>> this is not always the case. You don't provide an argument, you just
>> present your gut feeling as if it were the same thing as irrefutable
>> fact.
>
>
> I think Scott Aaronson has the right attitude toward this:
>
> https://www.scottaaronson.com/blog/?p=326

Thanks, interesting point by Aaronson. I will make a mental note of it
and move on :)
To be clear, I am not claiming "MWI is correct". I don't know if it
is. Gun to my head, I would bet on it, that's all.
Libertarianism, as well as communism, are silver-bullet ideologies
that ignore large aspects of human nature.

Telmo.

>
> Brent
>
>
> --
> You received this message because you are subscribed to the Google Groups
> "Everything List" group.
> To unsubscribe from this group and stop receiving emails from it, send an
> email to everything-list+unsubscr...@googlegroups.com.
> To post to this group, send email to everything-list@googlegroups.com.
> Visit this group at https://groups.google.com/group/everything-list.
> For more options, visit https://groups.google.com/d/optout.

-- 
You received this message because you are subscribed to the Google Groups 
"Everything List" group.
To unsubscribe from this group and stop receiving emails from it, send an email 
to everything-list+unsubscr...@googlegroups.com.
To post to this group, send email to everything-list@googlegroups.com.
Visit this group at https://groups.google.com/group/everything-list.
For more options, visit https://groups.google.com/d/optout.


Re: Is the "bubble multi-verse" and "qm many-worlds" the same thing?

2018-06-18 Thread agrayson2000


On Monday, June 18, 2018 at 9:46:31 AM UTC, telmo_menezes wrote:
>
> On 18 June 2018 at 05:04,  > wrote: 
> > 
> > 
> > On Sunday, June 17, 2018 at 11:43:33 AM UTC, telmo_menezes wrote: 
> >> 
> >> On 17 June 2018 at 13:26,   wrote: 
> >> > 
> >> > 
> >> > On Sunday, June 17, 2018 at 10:15:05 AM UTC, Jason wrote: 
> >> >> 
> >> >> 
> >> >> 
> >> >> On Sun, Jun 17, 2018 at 12:12 AM,  wrote: 
> >> >>> 
> >> >>> 
> >> >>> 
> >> >>>  why do you prefer the MWI compared to the Transactional 
> >> >>> Interpretation? 
> >> >>> I see both as absurd. so I prefer to assume the wf is just 
> epistemic, 
> >> >>> and/or 
> >> >>> that we have some holes in the CI which have yet to be resolved. AG 
> >> >>> 
> >> >>> -- 
> >> >> 
> >> >> 
> >> >> 
> >> >> 1. It's the simplest theory: "MWI" is just the Schrodinger equation, 
> >> >> nothing else. (it doesn't say Schrodinger's equation only applies 
> >> >> sometimes, 
> >> >> or only at certain scales) 
> >> >> 
> >> >> 2. It explains more while assuming less (it explains the appearance 
> of 
> >> >> collapse, without having to assume it, thus is preferred by Occam's 
> >> >> razor) 
> >> >> 
> >> >> 3. Like every other successful physical theory, it is linear, 
> >> >> reversible 
> >> >> (time-symmetric), continuous, deterministic and does not require 
> faster 
> >> >> than 
> >> >> light influences nor retrocausalities 
> >> >> 
> >> >> 4. Unlike single-universe or epistemic interpretations, "WF is real" 
> >> >> with 
> >> >> MWI is the only way we know how to explain the functioning of 
> quantum 
> >> >> computers (now up to 51 qubits) 
> >> >> 
> >> >> 5. Unlike copenhagen-type theories, it attributes no special 
> physical 
> >> >> abilities to observers or measurement devices 
> >> >> 
> >> >> 6. Most of all, theories of everything that assume a reality 
> containing 
> >> >> all possible observers and observations lead directly to 
> >> >> laws/postulates of 
> >> >> quantum mechanics (see Russell Standish's Theory of Nothing, Chapter 
> 7 
> >> >> and 
> >> >> Appendix D). 
> >> >> 
> >> >> Given #6, we should revise our view. It is not MWI and QM that 
> should 
> >> >> convince us of many worlds, but rather the assumption of many worlds 
> >> >> (an 
> >> >> infinite and infinitely varied reality) that gives us, and explains 
> all 
> >> >> the 
> >> >> weirdness of QM. This should overwhelmingly convince us of MWI-type 
> >> >> everything theories over any single-universe interpretation of 
> quantum 
> >> >> mechanics, which is not only absurd, but completely devoid of 
> >> >> explanation. 
> >> >> With the assumption of a large reality, QM is made explainable and 
> >> >> understandable: as a theory of observation within an infinite 
> reality. 
> >> >> 
> >> >> Jason 
> >> > 
> >> > 
> >> > You forgot #7. It asserts multiple, even infinite copies of an 
> observer, 
> >> > replete with memories, are created when an observer does a simple 
> >> > quantum 
> >> > experiment. So IMO the alleged "cure" is immensely worse than the 
> >> > disease, 
> >> > CI, that is, just plain idiotic. AG 
> >> 
> >> It is important to make the distinction between our intuition and 
> >> common sense and actual formal reasoning. The former can guide the 
> >> latter very successfully, but the history of science teaches us that 
> >> this is not always the case. You don't provide an argument, you just 
> >> present your gut feeling as if it were the same thing as irrefutable 
> >> fact. 
> >> 
> >> Jason presented you with a series of claims that need to be addressed 
> >> if you wish to refute his argument. Ignoring them and just writing 
> >> "idiotic" is not a valid argument. 
> >> 
> >> I also think you are being fooled by your idea of "creating a 
> >> universe". If the MWI is correct, then when the worlds bifurcate, the 
> >> memories of the past all come from the same parent state. I would say 
> >> that the MWI invites one to model reality more as a tree of states. 
> >> There are no entire universes being created out of thin air in the way 
> >> you seem to suggest. 
> > 
> > 
> > So you agree that universes created out of thin air fail the smell test? 
> But 
> > that's what your words, your "branches" as it were, imply. AG 
>
> I have the impression that you insist on the classical view of 
> reality. If this view were correct, universes would have to be 
> "created out of thing air" for the MWI to work. In the sense that all 
> the actual particles and their properties would have to be physically 
> copied to create a new world where Joe the Plumber witnesses a certain 
> experimental outcome. This does not pass the "smell test" indeed, 
> because then one would have to explain the copying mechanism. But: 
>

*No "but's" about it! That's the 800 pound gorilla in the room that Many 
Worlder's studiously ignore. There's no proposed mechanism to explain the 
copying! They will appeal to the mathematics, which they rely on 
uncritically. But it's obvious that 

Re: Is the "bubble multi-verse" and "qm many-worlds" the same thing?

2018-06-18 Thread Jason Resch
On Mon, Jun 18, 2018 at 7:04 AM,  wrote:

>
>>
> *No "but's" about it! That's the 800 pound gorilla in the room that Many
> Worlder's studiously ignore. There's no proposed mechanism to explain the
> copying! They will appeal to the mathematics, which they rely on
> uncritically. But it's obvious that such reliance can lead to ridiculous
> results if taken literally. For example, Maxwell's equations have plane
> wave solutions, but plane waves don't exist! Think about what a plane wave
> is; all peaks and troughs extend to infinity, advancing along an infinite
> plane in every direction. And No, I don't have a classical view of reality,
> but neither do I fall in love with BS. AG*
>
>>
>>
>>
Rather thank think of it like copying, think of it like this:

- When a photon hits a semi-silvered mirror, it splits and takes two
paths.  Now, both the photon's position and its direction are multi-valued.
- We can use full-silvered mirrors to redirect this "multi-valued" photon
such that both of its positions will converge (while nonetheless traveling
in different directions on that intercept course)
- We can place an electron at that point of interception, such that this
multi-velocity photon interacts with and strikes this electron (which right
now only has one position and zero velocity)
- When the multi-velocity photon hits the electron, the electron now has
two velocities (you could view it as in one case, it was hit from below,
and in another case it was hit from the side).

So we see a clear case, where a photon, being in a super-position of
states, when it interacts with an electron, it's multiple states transfer
to that electron such that it now ends up in a super position, and will
have different positions and velocities over time.  Anything that interacts
with that electron, or doesn't interact with it will also become
multi-valued.

Let's say there is a detection screen to the right of the electron,  the
multi-valued electron interacts with the atoms in the detection screen.
Because the electron's position is multi-valued, and only one of the
electron's velocity's intercept the screen, the atoms in the detection
screen also enter a super position, of having interacted with the electron
and having not interacted with the electron (the upward moving electron
velocity misses the screen).  So now there is a superposition of states in
the room where a flash flash of light from the screen has occurred, and
where no flash of light on the screen has occurred.

You can continue this example forward until you end up with yourself in the
room with two different brains, one where it is encoding memories of having
seen a flash of light and another where it did not.  Nothing special is
itnroduced by the observer, the observer is, afterall, just a collection of
particles, each of which may or may not enter a superposition of states,
depending on whether or not they interact with any particle that is in a
super position.

This is how the "copying" works.  It's just state transfer.  Think of it as:

x is in a superposition of +3 and - 3.  In other words x = ±3
y is 5

y interacts with x as follows: y = y * x

What this means is y = 5 * (±3)

Now y is multi-valued, in a superposition of 15 and -15.

No magic full scale copying of entire universes.  It's just particle states
can be multi-valued, and anything that interacts with a multi-valued
particle is effected by it being multi-valued.

Jason

-- 
You received this message because you are subscribed to the Google Groups 
"Everything List" group.
To unsubscribe from this group and stop receiving emails from it, send an email 
to everything-list+unsubscr...@googlegroups.com.
To post to this group, send email to everything-list@googlegroups.com.
Visit this group at https://groups.google.com/group/everything-list.
For more options, visit https://groups.google.com/d/optout.


Re: Is the "bubble multi-verse" and "qm many-worlds" the same thing?

2018-06-18 Thread Jason Resch
On Sun, Jun 17, 2018 at 6:23 PM,  wrote:

>
>
> On Sunday, June 17, 2018 at 9:24:40 PM UTC, Jason wrote:
>>
>>
>>
>> On Sunday, June 17, 2018,  wrote:
>>
>>>
>>>
>>> On Sunday, June 17, 2018 at 12:29:35 PM UTC, Jason wrote:



 On Sun, Jun 17, 2018 at 6:26 AM,  wrote:

>
>
> On Sunday, June 17, 2018 at 10:15:05 AM UTC, Jason wrote:
>>
>>
>>
>> On Sun, Jun 17, 2018 at 12:12 AM,  wrote:
>>
>>>
>>>
>>> * why do you prefer the MWI compared to the Transactional
>>> Interpretation? I see both as absurd. so I prefer to assume the wf is 
>>> just
>>> epistemic, and/or that we have some holes in the CI which have yet to be
>>> resolved. AG *
>>>
>>> --
>>>
>>
>>
>> 1. It's the simplest theory: "MWI" is just the Schrodinger equation,
>> nothing else. (it doesn't say Schrodinger's equation only applies
>> sometimes, or only at certain scales)
>>
>> 2. It explains more while assuming less (it explains the appearance
>> of collapse, without having to assume it, thus is preferred by Occam's
>> razor)
>>
>> 3. Like every other successful physical theory, it is linear,
>> reversible (time-symmetric), continuous, deterministic and does not 
>> require
>> faster than light influences nor retrocausalities
>>
>> 4. Unlike single-universe or epistemic interpretations, "WF is real"
>> with MWI is the only way we know how to explain the functioning of 
>> quantum
>> computers (now up to 51 qubits)
>>
>> 5. Unlike copenhagen-type theories, it attributes no special physical
>> abilities to observers or measurement devices
>>
>> 6. Most of all, theories of everything that assume a reality
>> containing all possible observers and observations lead directly to
>> laws/postulates of quantum mechanics (see Russell Standish's Theory
>> of Nothing ,
>> Chapter 7 and Appendix D).
>>
>> Given #6, we should revise our view. It is not MWI and QM that should
>> convince us of many worlds, but rather the assumption of many worlds (an
>> infinite and infinitely varied reality) that gives us, and *explains
>> *all the weirdness of QM. This should overwhelmingly convince us of
>> MWI-type everything theories over any single-universe interpretation of
>> quantum mechanics, which is not only absurd, but completely devoid of
>> explanation. With the assumption of a large reality, QM is made 
>> explainable
>> and understandable: as a theory of observation within an infinite 
>> reality.
>>
>> Jason
>>
>
> *You forgot #7. It asserts multiple, even infinite copies of an
> observer, replete with memories, are created when an observer does a 
> simple
> quantum experiment. So IMO the alleged "cure" is immensely worse than the
> disease, CI, that is, just plain idiotic. AG *
>
>
>
 There are many atoms, many planets, many solar systems, many galaxies,
 many Hubble volumes, and it is believed many universes.  On what basis are
 you so certain there aren't many histories? (That is, other states in the
 wave function that are predicted to be there by our well established
 scientific theories, but which the theory explains we cannot see or
 interact with except in very limited controlled manners)?

 If you find MWI distasteful you might prefer to think of it as the
 many-minds interpretation as described by Heinz-Dieter Zeh, or the
 "zero-universe interpretation" as explained by Ron Garrett:
 https://www.youtube.com/watch?v=dEaecUuEqfc

 I think you are hung up on the "creation", I think it is conceptually
 easier to grasp under the understanding that it is all already there.  If
 you look at the homepage of Wei Dai (who founded this e-mailing list
  20 years ago) he outlines what
 he calls "a very simple interpretation of quantum mechanics
 " which is basically
 this: all the states are already there.

>>>
>>> *Sounds like Super-Determinism proposed by t'Hooft, and referenced
>>> yesterday by Brent, which proposes the universe knows beforehand what kind
>>> of experiment Joe the Plumber will perform. Too ridiculous for my tastes,
>>> and of course untestable. IMO, one of the "achievements" of quantum theory
>>> is to make otherwise intelligent persons totally gullible in what they
>>> believe as plausible.  AG*
>>>


>> I agree with you about super derterminism being too ridiculous to
>> believe. But super derterminism is a different animal from "block time".
>> Super derterminism is the idea that the universe conspires against all
>> experimenters and knows what they will measure before they measure it, and
>> chooses values they will measure to 

Re: Do we live within a Diophantine equation?

2018-06-18 Thread Brent Meeker
How is that any different than simply saying they are computable to 
arbitrary accuracy, in the Church-Turing sense.


Brent

On 6/17/2018 3:32 PM, Lawrence Crowell wrote:
The Schrödinger equation is integrable for completely unitary systems. 
This hold for Hamiltonians that are complex or nonlinear, where 
perturbation methods are often used. My thought is that all possible 
quantum eigenvalues are computable with Diophantine equations, where 
some Gödel numbering scheme from the solutions to the actual numbers 
can exist.


LC

On Sunday, June 17, 2018 at 3:42:56 PM UTC-5, Jason wrote:

Hi Lawrence,

Is the evolution of states of the wave function computable? If so
then the result of MRDP implies it is Diophantine.

Jason

On Sunday, June 17, 2018, Lawrence Crowell
> wrote:

I have Matiyasevich's paper on the MRDP theorem. I have not as
yet read it. I have had this idea that a general scheme for
quantum eigenvalues could by Diophantine. This would then be a
sort of universal dovetailer of all possible physical states.
Unfortunately this is an area I have thought about some, but
as yet have never endeavored to pursue.

LC

On Saturday, June 16, 2018 at 7:18:41 PM UTC-5, Jason wrote:

In solving Hilbert's 10th problem
 in
the negative, the work of Martin Davis, Yuri Matiyasevich,
Hilary Putnam and Julia Robinson culminated in 1970 with
the MRDP theorem


which concludes:

*Every computably enumerable set has a representation as a
Diophantine equation
 (an
equation involving only integer coefficients and variables).*

This shocked number theorists, because it meant simple
equations involving nothing more than a few integer
variables have the full power of Turing machines. In fact,
it was shown by Yuri Matiyasevich that a universal
Diophantine equation can be made with as few as 9 unknowns.

Some examples:

  * k is even if there exists a solution to: k - 2x = 0
  * k is a perfect square if there exists a solution to: k
- x^2 = 0
  * k is a Fibonacci number if there exists a solution to:
k^4 - k^2*x^2 - x^4 - 1 = 0
  * (k+2) is a prime number if there exists a solution to
the sum of: (these 14 equations
)
  * k is a LISP program having output n, if the equation
having variables: k, n, x1, x2, x3 ... x2 (a
polynomial having ~20,000 variables
) has a solution.

The universality of Diophantine equations means there are
polynomial equations that compute things quite surprising,
such as polynomials that have solutions of 0, IFF:

  * One of the variables "k" is a valid MP3 file.
  * One of the variables "k" is a JPEG image containing
the image of a cat (where the equation implements the
same computation as a neural network trained to
recognize images of cats)
  * For two of the variables "y" and "x", "y" equals a
state of a chess board after deep blue makes a move
given a chess board with a state of "x".
  * For two of the variables "y" and "x", "y" equals the
state of the Universal Dovetailer after performing "n"
steps of execution.


The last example seems to suggest to me, that pure
arithmetical truth, concerning the solutions to equations,
is identical to computation.  That is to say, certain
mathematical statements carry with them (effectively)
Turing machines, and their executions.

Just as all solutions to the deep-blue implementing
equation is equivalent to the computations that Deep blue
makes when evaluating the board, and all solutions to the
cat recognizing equation are equivalent to the processing
done by the trained neural network, all solutions to the
LISP equation are equivalent to the execution of every
possible LISP program (including the UD).

Does this our conscious experience might be a direct
consequence of Diophantine equations?

Can Diophantine equations for a single set of parameters
model non-halting programs like the UD, or one must
consider the set of of 

RE: Mathematics as the result of natural selection

2018-06-18 Thread Steven Ridgway
On Mon, Jun 18, 2018 at 01:25 Dr Russell Standish wrote:
> "But presumably the argument is about certain cognitive skills which 
helped our species be extraordinarily successful, and also gave us the 
capability to understand algebraic topology."

I've always found it a bit mysterious that humans are so good at abstract 
mathematics. I can see that the evolutionary pressures to improve tool making 
and hunting skills could have given us basic mathematical capabilities - but we 
are far better at it than seems reasonable. i.e. it seems a stretch to imagine 
our ability to understand differential equations and prove Fermat's last 
theorem just fell into place as an accidental by product of something else.

It seems to me that a lot of complex engineering in our brains must exist to 
support the level of abstract reasoning we are capable of - and I don't see 
much evolutionary advantage to explain how this evolved.

We are familiar with the idea that a large multiverse could explain the 
apparent fine tuning of our universe to support conscious observers. I.e. given 
we are conscious observers it shouldn't be surprising that we find ourselves in 
a part of the multiverse that allows our existence.

However, right now we aren't just conscious observers, we are conscious 
observers pondering the unreasonable effectiveness of brains to do mathematics. 
Maybe similarly to the fine tuning argument we shouldn't be surprised to find 
ourselves in a part of the multiverse where brains did develop mathematical 
ability. It would have been extremely unlikely for our brains to have evolved 
the way they did - but in a sufficiently large multiverse we will inevitably 
find ourselves in the place where it did - given that we are observer moments 
that must have exactly that kind of abstract reasoning capability to understand 
this point!

Is it valid to use this kind of reasoning? To use the details of the type of 
conscious experience we are having right now to condition the type of universe 
we expect to find ourselves in? I'm not sure to be honest - but I think there 
is a mystery to be explained so the idea is appealing.

Note if it's true that evolving mathematical capability was a long shot, then a 
consequence of it would be that it would be very unlikely that we find 
technologically advanced aliens in the observable universe. There are a lot of 
stars out there - but the small probability of brains evolving abstract 
reasoning would overwhelm that I suspect.

- Steven Ridgway




-- 


Dr Russell StandishPhone 0425 253119 (mobile)
Principal, High Performance Coders
Visiting Senior Research Fellowhpco...@hpcoders.com.au
Economics, Kingston University http://www.hpcoders.com.au


--
You received this message because you are subscribed to the Google Groups 
"Everything List" group.
To unsubscribe from this group and stop receiving emails from it, send an email 
to everything-list+unsubscr...@googlegroups.com.
To post to this group, send email to everything-list@googlegroups.com.
Visit this group at https://groups.google.com/group/everything-list.
For more options, visit https://groups.google.com/d/optout.

-- 
You received this message because you are subscribed to the Google Groups 
"Everything List" group.
To unsubscribe from this group and stop receiving emails from it, send an email 
to everything-list+unsubscr...@googlegroups.com.
To post to this group, send email to everything-list@googlegroups.com.
Visit this group at https://groups.google.com/group/everything-list.
For more options, visit https://groups.google.com/d/optout.


Re: Is the "bubble multi-verse" and "qm many-worlds" the same thing?

2018-06-18 Thread Jason Resch
On Sun, Jun 17, 2018 at 6:37 PM, Russell Standish 
wrote:

> On Sun, Jun 17, 2018 at 07:20:10AM -0500, Jason Resch wrote:
> > >
> > >
> > > 6. Most of all, theories of everything that assume a reality containing
> > > all possible observers and observations lead directly to
> laws/postulates of
> > > quantum mechanics (see Russell Standish's Theory of Nothing
> > > , Chapter 7 and
> > > Appendix D).
> > >
> > >
> > > Unfortunately, Russell's attempt to derive quantum mechanics from the
> > > plenum of all possible bit strings failed at the first step. So you
> don't
> > > have much support from this.
> > >
> >
> > I would be very interested to see this, do you recall the subject or time
> > frame of this discussion?
>
> The subject thread was "Do Observer Moments form a Vecor Space?". The
> misspelling of Vector might help find the thread.
>

Thank you! I will catch up on this thread.


>
> Actually, "failing at the first step" was not my recollection of the
> discussion. Bruce had some important critiques, the most important of
> which is that the linear span of projected states L(ψₐ) that is used
> for defining the inner product (D.9) is, well, as arbitrary as the
> originally chosen observable A.
>
> My first cut at an answer to this tried to identify the everything
> with the origin of the vector space - which had a nice property that
> a complex field was required in order for some non-everything states
> not to collapse to the origin. However, that approach ultimately ran
> into a problem with non-orthonormalisability of the basis states. So I
> tried an alternate approach with the everything being a primitive
> direction in the vector space, and all subsets of the everything being
> orthogonal to it. This had some very nice properties, including the
> subset measure being given by the square of the vector norm, and that
> the vector space is a vector (or spectral) measure, the most general
> kind of measure there is. But lost was the nice requirement for the
> field to be complex, which was always a bit of a problem with the
> original derivation.
>
> I've been meaning to get this in publishable form, but time and other
> commitments have gotten in my way.
>
>
I (and I know many others on this list and elsewhere) eagerly await and
look forward this.


> In the meantime, Bruce thought he had a proof this was impossible to
> do (ie a vector space representation of the powerset of bitstrings that
> gives rise to the Born rule). However, he has yet to present his
> proof. My work mentioned above, appears to be a counterexample.
>
> In the meantime, another problem came to my attention from Markus
> Mueller (arxiv:1712.0181), where he points out that it is an open
> question whether transition probability for process on strings is
> naturally Markovian. The latter portion of my proof, in particular
> (D.13) is assuming a Markovian process.
>
>

Thanks for the reference, I will check it out.

Might combining your theory with a theory of computation (like the UDA) be
helpful in linking or otherwise tying together successively observed bit
strings?

I found the book "Trespassing on Einstein's Lawn
"
to be quite remarkable in breaking down the laws of physics to being the
bare minimum that is necessary to ensure consistency between observers. It
might be a fruitful avenue to explore, as it seems at least possibly
related to your effort.

Jason

-- 
You received this message because you are subscribed to the Google Groups 
"Everything List" group.
To unsubscribe from this group and stop receiving emails from it, send an email 
to everything-list+unsubscr...@googlegroups.com.
To post to this group, send email to everything-list@googlegroups.com.
Visit this group at https://groups.google.com/group/everything-list.
For more options, visit https://groups.google.com/d/optout.


Re: Mathematics as the result of natural selection

2018-06-18 Thread Jason Resch
I think a lot of our abstract reasoning ability results from our being
social creatures, and having to create mental models of other
people/groups/tribes, etc. to predict their behaviors under different
scenarios. To guess what they want, what they will do, what is likely to
happen if this happens or if that happens.  In our evolutionary
environment, nothing was more complex than other humans or groups of
humans, and the smarter we became, the smarter we had to get to maintain
some ability to model and predict the behavior of others.

It is then, perhaps not too major of a leap to turn this "abstract modeling
of a systems behavior" ability from analyzing people or groups, to
analyzing other systems, be they games, puzzles, engineering, mathematical
objects, contemplating physical laws, etc.

A question might arise, why don't other social animals have similar
abstract reasoning abilities?  Perhaps they do and cannot communicate it,
or perhaps communication itself adds so many additional layers of
complexity to the analyzing of social systems and people that it required
the evolution of special purpose structures in the brain which enhanced
abstract reasoning abilities.  Still a third option, is that human
analytical capability largely relies on the high level of language
processing capacity of the brain as a necessary ingredient in performing
some forms of abstract reasoning. -- I think there are exceptions and
counter examples in many of these cases, for example Tesla could visually
manipulate designs in his mind, and high level Chess players can see and
manipulate board states in their minds without relying on language to
represent those states.

Jason

On Mon, Jun 18, 2018 at 6:44 AM, Steven Ridgway  wrote:

> On Mon, Jun 18, 2018 at 01:25 Dr Russell Standish wrote:
> > "But presumably the argument is about certain cognitive skills
> which helped our species be extraordinarily successful, and also gave us
> the capability to understand algebraic topology."
>
> I've always found it a bit mysterious that humans are so good at abstract
> mathematics. I can see that the evolutionary pressures to improve tool
> making and hunting skills could have given us basic mathematical
> capabilities - but we are far better at it than seems reasonable. i.e. it
> seems a stretch to imagine our ability to understand differential equations
> and prove Fermat's last theorem just fell into place as an accidental by
> product of something else.
>
> It seems to me that a lot of complex engineering in our brains must exist
> to support the level of abstract reasoning we are capable of - and I don't
> see much evolutionary advantage to explain how this evolved.
>
> We are familiar with the idea that a large multiverse could explain the
> apparent fine tuning of our universe to support conscious observers. I.e.
> given we are conscious observers it shouldn't be surprising that we find
> ourselves in a part of the multiverse that allows our existence.
>
> However, right now we aren't just conscious observers, we are conscious
> observers pondering the unreasonable effectiveness of brains to do
> mathematics. Maybe similarly to the fine tuning argument we shouldn't be
> surprised to find ourselves in a part of the multiverse where brains did
> develop mathematical ability. It would have been extremely unlikely for our
> brains to have evolved the way they did - but in a sufficiently large
> multiverse we will inevitably find ourselves in the place where it did -
> given that we are observer moments that must have exactly that kind of
> abstract reasoning capability to understand this point!
>
> Is it valid to use this kind of reasoning? To use the details of the type
> of conscious experience we are having right now to condition the type of
> universe we expect to find ourselves in? I'm not sure to be honest - but I
> think there is a mystery to be explained so the idea is appealing.
>
> Note if it's true that evolving mathematical capability was a long shot,
> then a consequence of it would be that it would be very unlikely that we
> find technologically advanced aliens in the observable universe. There are
> a lot of stars out there - but the small probability of brains evolving
> abstract reasoning would overwhelm that I suspect.
>
> - Steven Ridgway
>
>
>
>
> --
>
> 
> 
> Dr Russell StandishPhone 0425 253119 (mobile)
> Principal, High Performance Coders
> Visiting Senior Research Fellowhpco...@hpcoders.com.au
> Economics, Kingston University http://www.hpcoders.com.au
> 
> 
>
> --
> You received this message because you are subscribed to the Google Groups
> "Everything List" group.
> To unsubscribe from this group and stop receiving emails from it, send an
> email to everything-list+unsubscr...@googlegroups.com.
> To post to 

Re: Is the "bubble multi-verse" and "qm many-worlds" the same thing?

2018-06-18 Thread Telmo Menezes
On 18 June 2018 at 05:04,   wrote:
>
>
> On Sunday, June 17, 2018 at 11:43:33 AM UTC, telmo_menezes wrote:
>>
>> On 17 June 2018 at 13:26,   wrote:
>> >
>> >
>> > On Sunday, June 17, 2018 at 10:15:05 AM UTC, Jason wrote:
>> >>
>> >>
>> >>
>> >> On Sun, Jun 17, 2018 at 12:12 AM,  wrote:
>> >>>
>> >>>
>> >>>
>> >>>  why do you prefer the MWI compared to the Transactional
>> >>> Interpretation?
>> >>> I see both as absurd. so I prefer to assume the wf is just epistemic,
>> >>> and/or
>> >>> that we have some holes in the CI which have yet to be resolved. AG
>> >>>
>> >>> --
>> >>
>> >>
>> >>
>> >> 1. It's the simplest theory: "MWI" is just the Schrodinger equation,
>> >> nothing else. (it doesn't say Schrodinger's equation only applies
>> >> sometimes,
>> >> or only at certain scales)
>> >>
>> >> 2. It explains more while assuming less (it explains the appearance of
>> >> collapse, without having to assume it, thus is preferred by Occam's
>> >> razor)
>> >>
>> >> 3. Like every other successful physical theory, it is linear,
>> >> reversible
>> >> (time-symmetric), continuous, deterministic and does not require faster
>> >> than
>> >> light influences nor retrocausalities
>> >>
>> >> 4. Unlike single-universe or epistemic interpretations, "WF is real"
>> >> with
>> >> MWI is the only way we know how to explain the functioning of quantum
>> >> computers (now up to 51 qubits)
>> >>
>> >> 5. Unlike copenhagen-type theories, it attributes no special physical
>> >> abilities to observers or measurement devices
>> >>
>> >> 6. Most of all, theories of everything that assume a reality containing
>> >> all possible observers and observations lead directly to
>> >> laws/postulates of
>> >> quantum mechanics (see Russell Standish's Theory of Nothing, Chapter 7
>> >> and
>> >> Appendix D).
>> >>
>> >> Given #6, we should revise our view. It is not MWI and QM that should
>> >> convince us of many worlds, but rather the assumption of many worlds
>> >> (an
>> >> infinite and infinitely varied reality) that gives us, and explains all
>> >> the
>> >> weirdness of QM. This should overwhelmingly convince us of MWI-type
>> >> everything theories over any single-universe interpretation of quantum
>> >> mechanics, which is not only absurd, but completely devoid of
>> >> explanation.
>> >> With the assumption of a large reality, QM is made explainable and
>> >> understandable: as a theory of observation within an infinite reality.
>> >>
>> >> Jason
>> >
>> >
>> > You forgot #7. It asserts multiple, even infinite copies of an observer,
>> > replete with memories, are created when an observer does a simple
>> > quantum
>> > experiment. So IMO the alleged "cure" is immensely worse than the
>> > disease,
>> > CI, that is, just plain idiotic. AG
>>
>> It is important to make the distinction between our intuition and
>> common sense and actual formal reasoning. The former can guide the
>> latter very successfully, but the history of science teaches us that
>> this is not always the case. You don't provide an argument, you just
>> present your gut feeling as if it were the same thing as irrefutable
>> fact.
>>
>> Jason presented you with a series of claims that need to be addressed
>> if you wish to refute his argument. Ignoring them and just writing
>> "idiotic" is not a valid argument.
>>
>> I also think you are being fooled by your idea of "creating a
>> universe". If the MWI is correct, then when the worlds bifurcate, the
>> memories of the past all come from the same parent state. I would say
>> that the MWI invites one to model reality more as a tree of states.
>> There are no entire universes being created out of thin air in the way
>> you seem to suggest.
>
>
> So you agree that universes created out of thin air fail the smell test? But
> that's what your words, your "branches" as it were, imply. AG

I have the impression that you insist on the classical view of
reality. If this view were correct, universes would have to be
"created out of thing air" for the MWI to work. In the sense that all
the actual particles and their properties would have to be physically
copied to create a new world where Joe the Plumber witnesses a certain
experimental outcome. This does not pass the "smell test" indeed,
because then one would have to explain the copying mechanism. But:

For a long time there has been overwhelming experimental evidence that
the building blocks of reality are not akin to billiard balls
colliding. They are something else, something that necessarily we
cannot make sense of by applying our experience of reality at our
level of observation. Objects that we directly observe are not in two
different states at the same time, nor do they interfere with
themselves to create waves. So any hope of coming up with an
explanation that does not feel weird to us is far away in the
rear-view mirror.

Many people, me included (not that that is worth much, I'm not a
physicist), feel that the MWI is the simplest explanation so far. It
posits 

Re: Is the "bubble multi-verse" and "qm many-worlds" the same thing?

2018-06-18 Thread agrayson2000


On Monday, June 18, 2018 at 12:30:02 PM UTC, Jason wrote:
>
>
>
> On Mon, Jun 18, 2018 at 7:04 AM, > 
> wrote:
>
>>
>>>
>> *No "but's" about it! That's the 800 pound gorilla in the room that Many 
>> Worlder's studiously ignore. There's no proposed mechanism to explain the 
>> copying! They will appeal to the mathematics, which they rely on 
>> uncritically. But it's obvious that such reliance can lead to ridiculous 
>> results if taken literally. For example, Maxwell's equations have plane 
>> wave solutions, but plane waves don't exist! Think about what a plane wave 
>> is; all peaks and troughs extend to infinity, advancing along an infinite 
>> plane in every direction. And No, I don't have a classical view of reality, 
>> but neither do I fall in love with BS. AG*
>>
>>>
>>>
>>>
> Rather thank think of it like copying, think of it like this:
>
> - When a photon hits a semi-silvered mirror, it splits and takes two 
> paths.  Now, both the photon's position and its direction are multi-valued.
>


*Does it split into two photons, each having the same energy as the 
original photon? If so, where does the added energy come from. Or does it 
split in half? Or does it take two possible paths, not both paths 
simultaneously? TIA, AG *

> - We can use full-silvered mirrors to redirect this "multi-valued" photon 
> such that both of its positions will converge (while nonetheless traveling 
> in different directions on that intercept course)
> - We can place an electron at that point of interception, such that this 
> multi-velocity photon interacts with and strikes this electron (which right 
> now only has one position and zero velocity)
> - When the multi-velocity photon hits the electron, the electron now has 
> two velocities (you could view it as in one case, it was hit from below, 
> and in another case it was hit from the side).
>
> So we see a clear case, where a photon, being in a super-position of 
> states, when it interacts with an electron, it's multiple states transfer 
> to that electron such that it now ends up in a super position, and will 
> have different positions and velocities over time.  Anything that interacts 
> with that electron, or doesn't interact with it will also become 
> multi-valued.
>
> Let's say there is a detection screen to the right of the electron,  the 
> multi-valued electron interacts with the atoms in the detection screen. 
> Because the electron's position is multi-valued, and only one of the 
> electron's velocity's intercept the screen, the atoms in the detection 
> screen also enter a super position, of having interacted with the electron 
> and having not interacted with the electron (the upward moving electron 
> velocity misses the screen).  So now there is a superposition of states in 
> the room where a flash flash of light from the screen has occurred, and 
> where no flash of light on the screen has occurred.
>
> You can continue this example forward until you end up with yourself in 
> the room with two different brains, one where it is encoding memories of 
> having seen a flash of light and another where it did not.  Nothing special 
> is itnroduced by the observer, the observer is, afterall, just a collection 
> of particles, each of which may or may not enter a superposition of states, 
> depending on whether or not they interact with any particle that is in a 
> super position.
>
> This is how the "copying" works.  It's just state transfer.  Think of it 
> as:
>
> x is in a superposition of +3 and - 3.  In other words x = ±3
> y is 5
>
> y interacts with x as follows: y = y * x
>
> What this means is y = 5 * (±3)
>
> Now y is multi-valued, in a superposition of 15 and -15.
>
> No magic full scale copying of entire universes.  It's just particle 
> states can be multi-valued, and anything that interacts with a multi-valued 
> particle is effected by it being multi-valued.
>
> Jason
>

-- 
You received this message because you are subscribed to the Google Groups 
"Everything List" group.
To unsubscribe from this group and stop receiving emails from it, send an email 
to everything-list+unsubscr...@googlegroups.com.
To post to this group, send email to everything-list@googlegroups.com.
Visit this group at https://groups.google.com/group/everything-list.
For more options, visit https://groups.google.com/d/optout.


Re: Is the "bubble multi-verse" and "qm many-worlds" the same thing?

2018-06-18 Thread Jason Resch
On Sun, Jun 17, 2018 at 8:54 PM,  wrote:

>
>
>> *​>​ **I have always regarded Bell results as paradoxical, or if you
>>> prefer unintelligible,*
>>>
>>
>> It's not paradoxical because its not self contradictory and its not
>> unintelligible because the results are clear as a Bell (pun intended), they
>> are just very very odd.
>>
>
> *Do us all a favor and stop playing word games. What you call "odd", can
> easily been seen as paradoxical or unintelligible insofar as it can't be
> understood in terms of how we perceive space, or spatial extent, and of
> course causality AG *
>
>>
>>
>>> ​
>>>
>>
While definitely not obvious (it was missed by Bell and Einstein after
all), I think the result of Bell can be summarized into something quite
clear:

We must give up at least one of:
1. Special relativity
2. Quantum mechanics
3. A single universe

We can't keep all three.  I'll give up a single universe, as the
experimental evidence for 1 & 2 are quite strong. I have seen no evidence
in support of 3.

Jason

-- 
You received this message because you are subscribed to the Google Groups 
"Everything List" group.
To unsubscribe from this group and stop receiving emails from it, send an email 
to everything-list+unsubscr...@googlegroups.com.
To post to this group, send email to everything-list@googlegroups.com.
Visit this group at https://groups.google.com/group/everything-list.
For more options, visit https://groups.google.com/d/optout.


Re: Do we live within a Diophantine equation?

2018-06-18 Thread Lawrence Crowell
The Davis, Matiyasevich, Putnam, Robinson (DMPR) theorem proves that the 
solutions for any general element of a Diophantine set is Turing halting, 
but that any other element may not be. This means the solutions to 
Diophantine equations are recursively enumerable, and there is a Gödel 
theorem aspect to this. Now if we have some scheme for Gödel numbering 
quantum eigenvalues gn(λ) → P(a, x_1, x_2, ..., x_n), for λ an eigenvalue 
with a code mapped to the solution of a Diophantine equation. 

The non-solutions may then be the emergence of classicality. Quantum 
physics does not predict chaotic behavior, and chaotic behavior is in 
principle an endless recursion of orbits and "filigree" that is recursively 
enumerable. This may then be a way to think about the relationship between 
quantum mechanics and the emergence of classical physics with einselection.

LC

On Monday, June 18, 2018 at 6:46:17 AM UTC-5, Brent wrote:
>
> How is that any different than simply saying they are computable to 
> arbitrary accuracy, in the Church-Turing sense.
>
> Brent
>
> On 6/17/2018 3:32 PM, Lawrence Crowell wrote:
>
> The Schrödinger equation is integrable for completely unitary systems. 
> This hold for Hamiltonians that are complex or nonlinear, where 
> perturbation methods are often used. My thought is that all possible 
> quantum eigenvalues are computable with Diophantine equations, where some 
> Gödel numbering scheme from the solutions to the actual numbers can exist. 
>
> LC
>
> On Sunday, June 17, 2018 at 3:42:56 PM UTC-5, Jason wrote: 
>>
>> Hi Lawrence, 
>>
>> Is the evolution of states of the wave function computable? If so then 
>> the result of MRDP implies it is Diophantine. 
>>
>> Jason
>>
>> On Sunday, June 17, 2018, Lawrence Crowell  
>> wrote:
>>
>>> I have Matiyasevich's paper on the MRDP theorem. I have not as yet read 
>>> it. I have had this idea that a general scheme for quantum eigenvalues 
>>> could by Diophantine. This would then be a sort of universal dovetailer of 
>>> all possible physical states. Unfortunately this is an area I have thought 
>>> about some, but as yet have never endeavored to pursue. 
>>>
>>> LC
>>>
>>> On Saturday, June 16, 2018 at 7:18:41 PM UTC-5, Jason wrote: 

 In solving Hilbert's 10th problem 
  in the 
 negative, the work of Martin Davis, Yuri Matiyasevich, Hilary Putnam and 
 Julia Robinson culminated in 1970 with the MRDP theorem 
  
 which concludes:

 *Every computably enumerable set has a representation as a Diophantine 
 equation  (an equation 
 involving only integer coefficients and variables).*

 This shocked number theorists, because it meant simple equations 
 involving nothing more than a few integer variables have the full power of 
 Turing machines.  In fact, it was shown by Yuri Matiyasevich that a 
 universal Diophantine equation can be made with as few as 9 unknowns.

 Some examples:

- k is even if there exists a solution to: k - 2x = 0 
- k is a perfect square if there exists a solution to: k - x^2 = 0 
- k is a Fibonacci number if there exists a solution to: k^4 - 
k^2*x^2 - x^4 - 1 = 0 
- (k+2) is a prime number if there exists a solution to the sum of: 
(these 14 equations 
) 
- k is a LISP program having output n, if the equation having 
variables: k, n, x1, x2, x3 ... x2 (a polynomial having ~20,000 
variables ) has a solution. 

 The universality of Diophantine equations means there are polynomial 
 equations that compute things quite surprising, such as polynomials that 
 have solutions of 0, IFF:

- One of the variables "k" is a valid MP3 file. 
- One of the variables "k" is a JPEG image containing the image of 
a cat (where the equation implements the same computation as a neural 
network trained to recognize images of cats) 
- For two of the variables "y" and "x", "y" equals a state of a 
chess board after deep blue makes a move given a chess board with a 
 state 
of "x". 
- For two of the variables "y" and "x", "y" equals the state of the 
Universal Dovetailer after performing "n" steps of execution. 


 The last example seems to suggest to me, that pure arithmetical truth, 
 concerning the solutions to equations, is identical to computation.  That 
 is to say, certain mathematical statements carry with them (effectively) 
 Turing machines, and their executions.

 Just as all solutions to the deep-blue implementing equation is 
 equivalent to the computations that Deep 

Re: Is the "bubble multi-verse" and "qm many-worlds" the same thing?

2018-06-18 Thread Bruce Kellett

From: *Jason Resch* mailto:jasonre...@gmail.com>>


On Sun, Jun 17, 2018 at 6:24 PM, Bruce Kellett 
mailto:bhkell...@optusnet.com.au>> wrote:



Maybe it just means that we don't yet fully understand the
collapse. There are plenty of possibilities that don't resort to
magic.


I agree.

But what facts about our observations of collapse are not already 
fully explained by Decoherence?


Decoherence does not explain the transition from FAPP orthogonality to 
full orthogonality of the branch states. In other words, decoherence is 
unitary, so cannot explain the non-unitary trace over unobserved 
environmental entanglements inherent in the projection of actual 
experimental results.


In other words, what is left to solve about it? I thought Decoherence 
solved this (back in 1952 with Bohm).


Decoherence was not introduced by Bohm. The idea originated with Dieter 
Zeh in around 1974, if I remember correctly.



When you say non-local what type of non-locality do you mean?  It
is a local theory in the sense that physical objects interact
only with other physical objects in their proximity, and carry
information only at luminal or subluminal speeds.  See Q12 on
http://www.anthropic-principle.com/preprints/
manyworlds.html


Price's argument here has been shown to be invalid -- he
surreptitiously relies on non-locality.


Care to explain this non-locality and where it appears in a MWI 
explanation of the EPR paradox, for example?  I've provided 
explanations on this list before of how EPR/Bell operates under MWI 
without FTL influences.  So if you think they are required I would be 
interested to know where you think they appear and are necessary.


I have pointed out the flaw in Price's account previously on the list. 
Tipler makes the same mistake, as do several others. But rather that 
going through the argument here, I will postpone it to my discussion of 
your attempted local account. You make essentially the same mistake, so 
we can look at it then.


John Clark often says MWI is non local because the branches are not 
local to each other, but I think this is a redefinition of the common 
sense use of the term locality in physics. Is this what you mean by 
MWI being non local?


Not really, but John does have a point.

How do you explain the finite computational resources of a table-top 
quantum computer factoring a prime number in seconds when it would 
take a classical computer the size of the solar system 10^100 years to 
do the same calculation?


David Deutsch notes that quantum computers present a strong challenge 
to defenders of single-universe interpretations, saying “When a 
quantum computer delivers the output of such a computation, we shall 
know that those intermediate results must have been computed 
somewhere, because they were needed to produce the right answer. So I 
issue this challenge to those who still cling to a single-universe 
world view: if the universe we see around us is all there is, where 
are quantum computations performed? I have yet to receive a plausible 
reply.”



That might be Deutsch's opinion, but plenty of others think
differently. Quantum computers can easily be understood in a
single world account.


But it can't be explained in non-realist views of the wave function. 
For example, those that say it is nothing but a convenient tool for 
computing probabilities.


Why can't that account for quantum computing?

The reason is, here this "convenient tool" is computing results for us 
that we have no hope of ever computing ourselves.  How is something 
which isn't real, and isn't really there, yielding results of 
computations?


Quantum mechanics is weird!

You say others think differently, but don't allude to who those other 
thinkers are or what their thoughts are.  Do you have an explanation 
for quantum computers that works with the assumption the wave function 
is not real?


Yes. The particular person I was thinking of here is Scott Aaronson. He 
is no fan of Deutsh's approach to quantum computing and many worlds. He 
points out that quantum computers rely on interference between the 
components, and that is possible only in a single world.


What would you say about a conscious AI implemented on a quantum 
computer? Would it or would it not be capable of existing in and 
experiencing "many simulations"?


A quantum computer would be no different from a classical computer in so 
far as implementing AI is concerned. Why would you think it would be 
different?



[snip]



Bell had an implicit assumption in his reasoning, which is that only 
a single definite result is obtained by all parties for any given 
measurement (including those not performed).  This is not true under 
MWI so Bell's reasoning that there must be non-locality fails for 
MWI, as many-worlds lacks contra-factual definiteness.


Again from: 
http://www.anthropic-principle.com/preprints/manyworlds.html 

Re: Is the "bubble multi-verse" and "qm many-worlds" the same thing?

2018-06-18 Thread John Clark
On Mon, Jun 18, 2018 at 6:31 AM, Jason Resch  wrote:

​>* ​*
> *Superdeterminism says you and a remote partner could decide to use the
> digits of Pi to pseudorandomly select angles of measurement in a Bell
> experiment, then decide to use the digits of Euler's number. Yet somehow,
> the universe knew you and your friend had this agreement to use these
> digits of these constants, such that when it generated single pairs of
> photons, those photons would have just the right properties for QM
> statistics to not be violated. Also, the universe knew when you would
> decide to switch to use Euler's number, which perhaps was decided by the
> closing price of the stock market, all this information the universe knew
> and took into account*
>

And out of all the initial conditions the universe could have been in 13.8
billion years ago something placed it into the one initial condition that,
after 13.8 billion years of cosmic evolution, would cause the stock market
on planet earth to close in such a way that the two experimenters would
switch to using Euler's number and be fooled into questioning the validity
of locality and causality. I'll never be able to prove this idea is untrue
because the universe has conspired to make sure I'll never be able to think
of a way to do so. And maybe the Bible was right and God created the
universe only 6000 years ago complete with phony dinosaur bones already
buried in the ground and cosmic microwaves background radiation already in
the sky. I'll never be able to disprove this idea either  because being
omnipotent and omniscient God will always be able to fool me. Or maybe 6000
years is too long and the universe was created only 5 minutes ago complete
with memories of me being in the third grade. Maybe, but I sorta doubt it.

John K Clark

-- 
You received this message because you are subscribed to the Google Groups 
"Everything List" group.
To unsubscribe from this group and stop receiving emails from it, send an email 
to everything-list+unsubscr...@googlegroups.com.
To post to this group, send email to everything-list@googlegroups.com.
Visit this group at https://groups.google.com/group/everything-list.
For more options, visit https://groups.google.com/d/optout.


Re: Is the "bubble multi-verse" and "qm many-worlds" the same thing?

2018-06-18 Thread John Clark
On Mon, Jun 18, 2018 at 12:55 AM, Brent Meeker  wrote:

>JKC has mis-stated CI.  CI didn't say QM as embodied in the SWE was the
> theory of the world.  Bohr and Heisenberg both held that theclassical
> world was logically prior to the quantum


If so then forget string theory, Newton discovered the theory that fully
describes the fundamental underlying nature of reality way back in 1687.
And nobody knows what Copenhagen is saying. Bohr, Heisenberg, Wigner and
Wheeler all said they are ardent believers in the Copenhagen interpretation
but Wigner, a Nobel Prize winner, thought consciousness collapsed the
quantum wave function while Heisenberg, another Nobel prize winner,
insisted it did not; and that's a pretty big difference in my opinion. And
John Wheeler was Hugh Everett's thesis adviser and even wrote a letter to
Bohr that still exists saying that Many Worlds is fully consistent with the
Copenhagen Interpretation! As for Niels Bohr, he said "Never express
yourself more clearly than you are able to think” and as a result of that
to this day people are still arguing about what the man was trying to say.

So before we start debating its pros and cons we should figure out what the
hell the Copenhagen Interpretation is.

John K Clark

-- 
You received this message because you are subscribed to the Google Groups 
"Everything List" group.
To unsubscribe from this group and stop receiving emails from it, send an email 
to everything-list+unsubscr...@googlegroups.com.
To post to this group, send email to everything-list@googlegroups.com.
Visit this group at https://groups.google.com/group/everything-list.
For more options, visit https://groups.google.com/d/optout.


Re: Is the "bubble multi-verse" and "qm many-worlds" the same thing?

2018-06-18 Thread dlb


On Monday, June 18, 2018 at 12:41:57 AM UTC+2, Bruce wrote:
>
> From: dlb < bajl...@gmail.com 
>
>
> On Sunday, June 17, 2018 at 1:49:27 PM UTC+2, Bruce wrote: 
>>
>> From: dlb 
>>
>>
>> On Saturday, June 16, 2018 at 11:20:49 AM UTC+2, agrays...@gmail.com 
>> wrote: 
>>>
>>>
>>>
>>> *The single event outcome in this world is certain. We measure it. Those 
>>> in the other worlds are speculative at best, and DO give human beings the 
>>> power to create worlds. By choosing to do this or that quantum experiment, 
>>> worlds are created!  I'll ask you the same question I asked Clark, which he 
>>> has yet to answer. If you find mathematics to be the Gospel for truth, why 
>>> don't we observe advanced waves predicted by Maxwell's equations? AG *
>>>
>>   
>>
>> In short – interaction with the instrument of observation or some other 
>> absorber in the environment masks advanced waves:  The hidden arrow of 
>> electromagnetic radiation: unmasking advanced waves 
>> 
>>
>>
>> The Wheeler-Feynman absorber theory which postulated the equality of 
>> advanced and retarded waves in every electromagnetic interaction was shown 
>> to be non viable many years ago. Even Wheeler and Feynman abandoned the 
>> project. Cramer's retrocausality account of quantum mechanics has been 
>> falsified by the non-observation of the requisite advanced waves.
>>
>> Bruce
>>
>  
> According to Kastner, Wheeler was later advocating it again in 2003, and TI 
> is still perfectly viable: http://www.ijqf.org/archives/4794
>
>
> Kastner is one of the lone advocates for the transactional interpretation. 
> What new insight does she offer for the resurrection of the Wheeler-Feynman 
> absorber theory? I think Zeh has kill this fairly comprehensively in his 
> book "The Physical Basis of the Direction of Time". The TI has never been 
> viable because there is no evidence for advanced waves or the so-called 
> "offer waves", and there is no dynamical basis for the properties that such 
> waves are assumed to possess -- it all boils down to a particular magic.
>
> Bruce
>
>   

There is no evidence for advanced waves because almost no effort has been 
invested to detect advanced waves. From one rare attempt; Schmidt (1980) A 
search for advanced fields in electromagnetic radiation:


2.8 THE NECESSITY OF A DELIBERATE SEARCH


One may ask whether advanced effects, if they do 

exist, would have already been observed, in the course of

human experience. The presence of local absorber, whose

response would cancel advanced effects, is rarely avoided

inadvertently. In earth-based radio or light communications

the line connecting transmitter and receiver, when

extended in both directions, almost always intersects

absorber. Even if this were not the case there are two

reasons why advanced effects might not be seen without a

deliberate search " First, the advanced signal arrives

in such close time proximity to the retarded signal as

to require a deliberate effort to distinguish the two.

Second, the intensity of the advanced signal, as a fraction

of the retarded, is given by the fractional absence of

ultimate absorption. So, in the case of p a r t i a l

absorption, any advanced signal would appear attenuated

and might require deliberate signal processing techniques

(integration) to detect.

Finally, one must never forget the power with which

paradigms direct the work of scientists (Kuhn, 1970).

Anomalies encountered in work peripheral to an experiment

tend to be discounted unless they interfere with the goal

of the experiment.

All of these considerations argue for a deliberate

search for advanced effects.

 

Unfortunately, in his attempt to detect advanced waves Schmidt did not take 
into account the influence of the receiving antenna (a half-wave dipole) on 
the cancellation of advanced waves from the transmitting antenna (for that 
exact direction of radiation, as I pointed out in the paper I provided in 
my first post and in the reference 14, which is the reproduction of the 
Schmidt experiment in 2017 in the modified configuration, in which the 
influence of the receiving antenna was taken into account, as well as the 
influence of water vapor in the troposphere and absorption of interstellar 
hydrogen, which Schmidt also did not take into account).

-- 
You received this message because you are subscribed to the Google Groups 
"Everything List" group.
To unsubscribe from this group and stop receiving emails from it, send an email 
to everything-list+unsubscr...@googlegroups.com.
To post to this group, send email to everything-list@googlegroups.com.
Visit this group at https://groups.google.com/group/everything-list.
For more options, visit https://groups.google.com/d/optout.


Re: Is the "bubble multi-verse" and "qm many-worlds" the same thing?

2018-06-18 Thread John Clark
On Sun, Jun 17, 2018 at 9:54 PM,  wrote:

>> What about it, what is your theory of decoherence and how does it make
>> the CI less dumb?
>
>
> > *Not that I'm a great fan of decoherence theory, but it doest includes
> the apparatus, observer, and the rest of the environment in the measuring
> process.*


I know what quantum decoherence is but I've never heard of the term
"decoherence theory" before and I don't see how anyone can talk about
decoherence without referring to the rest of the environment. If X is the
only thing in the universe or the only thing that’s important then there is
nothing X can become cohered or de-cohered from.

*>>>  I have always regarded Bell results as paradoxical, or if you prefer
>>> unintelligible,*
>>
>>

>>It's not paradoxical because its not self contradictory and its not
>> unintelligible because the results are clear as a Bell (pun intended), they
>> are just very very odd.
>
>
> > *Do us all a favor and stop playing word games. *


Now you sound like Bruno. Do you really thing you can give words vague
ephemeral meanings and still do science?

*>What you call "odd", can easily been seen as paradoxical or
> unintelligible *


I don’t think we’re ever going to be able to figure out anything if the
meanings of words change at our whim from day to day. The reductio ad
absurdum proof is actually poorly named because it is not good enough to
show that a proposition will lead to something odd or even very odd to
prove that the proposition must be untrue, you’ve got to show it is
paradoxical, that is to say self contradictory. If Many World’s or the
Transactional Interpretation was true it would means some very odd things
were going on behind the scenes, but that doesn’t prove either of them is
untrue. In fact now that the Bell experiments have been performed one thing
we know for sure is that some very odd things ARE going on behind the
scenes, we’re just not sure exactly what they are. So if your explanation
is not very very odd it can’t be right.

I don’t know what it would mean if Copenhagen turned out to be true because
even those who say they believe in it can’t agree among themselves what
exactly the Copenhagen Interpretation is saying other than “shut up and
calculate”.

*> insofar as it can't be understood in terms of how we perceive space, or
> spatial extent, and of course causality *


So to have any hope of understanding what’s going on we’re going to have to
abandon comfortable concepts like causality and the normal way we perceive
space and embrace something much stranger.

>> Are you confusing plane waves with advanced waves?
>
>
> >*Definitely not. Plane waves don't exist except possibly in your
> imagination.  AG*


To a good approximation a Laser produces plane waves, the electromagnetic
wave fronts form parallel planes, that’s why a Laser beam spreads out very
little with distance unlike a light bulb which produces a spherical wave.
But I don’t see what this has to do with quantum interpretation and I don’t
know of any physicists who thinks plane waves are more controversial than
spherical waves.

John K Clark

-- 
You received this message because you are subscribed to the Google Groups 
"Everything List" group.
To unsubscribe from this group and stop receiving emails from it, send an email 
to everything-list+unsubscr...@googlegroups.com.
To post to this group, send email to everything-list@googlegroups.com.
Visit this group at https://groups.google.com/group/everything-list.
For more options, visit https://groups.google.com/d/optout.


Re: Is the "bubble multi-verse" and "qm many-worlds" the same thing?

2018-06-18 Thread Jason Resch
On Mon, Jun 18, 2018 at 7:42 AM,  wrote:

>
>
> On Monday, June 18, 2018 at 12:30:02 PM UTC, Jason wrote:
>>
>>
>>
>> On Mon, Jun 18, 2018 at 7:04 AM,  wrote:
>>
>>>

>>> *No "but's" about it! That's the 800 pound gorilla in the room that Many
>>> Worlder's studiously ignore. There's no proposed mechanism to explain the
>>> copying! They will appeal to the mathematics, which they rely on
>>> uncritically. But it's obvious that such reliance can lead to ridiculous
>>> results if taken literally. For example, Maxwell's equations have plane
>>> wave solutions, but plane waves don't exist! Think about what a plane wave
>>> is; all peaks and troughs extend to infinity, advancing along an infinite
>>> plane in every direction. And No, I don't have a classical view of reality,
>>> but neither do I fall in love with BS. AG*
>>>



>> Rather thank think of it like copying, think of it like this:
>>
>> - When a photon hits a semi-silvered mirror, it splits and takes two
>> paths.  Now, both the photon's position and its direction are multi-valued.
>>
>
>
> *Does it split into two photons, each having the same energy as the
> original photon? If so, where does the added energy come from. Or does it
> split in half? Or does it take two possible paths, not both paths
> simultaneously? TIA, AG*
>

Think of it being only a change to the photon's properties.  It's not
really two photons, it's a single photon, now holding multiple values for
some of its properties (which include velocity, position, frequency,
polarization, etc.)  After interacting with a semi-silvered mirror, the
same thing always happens to a photon, it takes on multiple values for its
velocity.

Jason



>
>
>> - We can use full-silvered mirrors to redirect this "multi-valued" photon
>> such that both of its positions will converge (while nonetheless traveling
>> in different directions on that intercept course)
>> - We can place an electron at that point of interception, such that this
>> multi-velocity photon interacts with and strikes this electron (which right
>> now only has one position and zero velocity)
>> - When the multi-velocity photon hits the electron, the electron now has
>> two velocities (you could view it as in one case, it was hit from below,
>> and in another case it was hit from the side).
>>
>> So we see a clear case, where a photon, being in a super-position of
>> states, when it interacts with an electron, it's multiple states transfer
>> to that electron such that it now ends up in a super position, and will
>> have different positions and velocities over time.  Anything that interacts
>> with that electron, or doesn't interact with it will also become
>> multi-valued.
>>
>> Let's say there is a detection screen to the right of the electron,  the
>> multi-valued electron interacts with the atoms in the detection screen.
>> Because the electron's position is multi-valued, and only one of the
>> electron's velocity's intercept the screen, the atoms in the detection
>> screen also enter a super position, of having interacted with the electron
>> and having not interacted with the electron (the upward moving electron
>> velocity misses the screen).  So now there is a superposition of states in
>> the room where a flash flash of light from the screen has occurred, and
>> where no flash of light on the screen has occurred.
>>
>> You can continue this example forward until you end up with yourself in
>> the room with two different brains, one where it is encoding memories of
>> having seen a flash of light and another where it did not.  Nothing special
>> is itnroduced by the observer, the observer is, afterall, just a collection
>> of particles, each of which may or may not enter a superposition of states,
>> depending on whether or not they interact with any particle that is in a
>> super position.
>>
>> This is how the "copying" works.  It's just state transfer.  Think of it
>> as:
>>
>> x is in a superposition of +3 and - 3.  In other words x = ±3
>> y is 5
>>
>> y interacts with x as follows: y = y * x
>>
>> What this means is y = 5 * (±3)
>>
>> Now y is multi-valued, in a superposition of 15 and -15.
>>
>> No magic full scale copying of entire universes.  It's just particle
>> states can be multi-valued, and anything that interacts with a multi-valued
>> particle is effected by it being multi-valued.
>>
>> Jason
>>
> --
> You received this message because you are subscribed to the Google Groups
> "Everything List" group.
> To unsubscribe from this group and stop receiving emails from it, send an
> email to everything-list+unsubscr...@googlegroups.com.
> To post to this group, send email to everything-list@googlegroups.com.
> Visit this group at https://groups.google.com/group/everything-list.
> For more options, visit https://groups.google.com/d/optout.
>

-- 
You received this message because you are subscribed to the Google Groups 
"Everything List" group.
To unsubscribe from this group and stop receiving emails 

Re: Is the "bubble multi-verse" and "qm many-worlds" the same thing?

2018-06-18 Thread Jason Resch
On Mon, Jun 18, 2018 at 7:38 AM, Bruce Kellett 
wrote:

> From: Jason Resch < jasonre...@gmail.com>
>
>
> On Sun, Jun 17, 2018 at 6:24 PM, Bruce Kellett <
> bhkell...@optusnet.com.au> wrote:
>
>>
>> Maybe it just means that we don't yet fully understand the collapse.
>> There are plenty of possibilities that don't resort to magic.
>>
>>
> I agree.
>
> But what facts about our observations of collapse are not already fully
> explained by Decoherence?
>
>
> Decoherence does not explain the transition from FAPP orthogonality to
> full orthogonality of the branch states.
>

Is it ever full/complete?  Isn't Decoherence only ever approximate?


> In other words, decoherence is unitary, so cannot explain the non-unitary
> trace over unobserved environmental entanglements inherent in the
> projection of actual experimental results.
>
>
You get unitarity when you couple Decoherence with many worlds.  No world
destroying magic is required to cause other universes to go to zero, and
our (how lucky we are) world to go to 1.

In other words, what is left to solve about it? I thought Decoherence
> solved this (back in 1952 with Bohm).
>
>
> Decoherence was not introduced by Bohm. The idea originated with Dieter
> Zeh in around 1974, if I remember correctly.
>

It was used in Bohemian mechanics, and later by Everett. See "'7. THE
ORIGIN OF THE STATISTICAL ENSEMBLE IN THE QUANTUM THEORY" of:
http://cqi.inf.usi.ch/qic/bohm2.pdf



>
>
> When you say non-local what type of non-locality do you mean?  It is a
>> local theory in the sense that physical objects interact only with other
>> physical objects in their proximity, and carry information only at luminal
>> or subluminal speeds.  See Q12 on  
>> http://www.anthropic-principle.com/preprints/manyworlds.html
>>
>>
>> Price's argument here has been shown to be invalid -- he surreptitiously
>> relies on non-locality.
>>
>>
> Care to explain this non-locality and where it appears in a MWI
> explanation of the EPR paradox, for example?  I've provided explanations on
> this list before of how EPR/Bell operates under MWI without FTL
> influences.  So if you think they are required I would be interested to
> know where you think they appear and are necessary.
>
>
> I have pointed out the flaw in Price's account previously on the list.
> Tipler makes the same mistake, as do several others. But rather that going
> through the argument here, I will postpone it to my discussion of your
> attempted local account. You make essentially the same mistake, so we can
> look at it then.
>
> John Clark often says MWI is non local because the branches are not local
> to each other, but I think this is a redefinition of the common sense use
> of the term locality in physics. Is this what you mean by MWI being non
> local?
>
>
> Not really, but John does have a point.
>
>  How do you explain the finite computational resources of a table-top
> quantum computer factoring a prime number in seconds when it would take a
> classical computer the size of the solar system 10^100 years to do the same
> calculation?
>
> David Deutsch notes that quantum computers present a strong challenge to
> defenders of single-universe interpretations, saying “When a quantum
> computer delivers the output of such a computation, we shall know that
> those intermediate results must have been computed somewhere, because they
> were needed to produce the right answer. So I issue this challenge to those
> who still cling to a single-universe world view: if the universe we see
> around us is all there is, where are quantum computations performed? I have
> yet to receive a plausible reply.”
>
>>
>> That might be Deutsch's opinion, but plenty of others think differently.
>> Quantum computers can easily be understood in a single world account.
>>
>>
> But it can't be explained in non-realist views of the wave function. For
> example, those that say it is nothing but a convenient tool for computing
> probabilities.
>
>
> Why can't that account for quantum computing?
>
> The reason is, here this "convenient tool" is computing results for us
> that we have no hope of ever computing ourselves.  How is something which
> isn't real, and isn't really there, yielding results of computations?
>
>
> Quantum mechanics is weird!
>
>
This is what David Deutsch was referring to when he says he has yet to
receive a plausible reply.

I think the answer is, however, rather clear if you only consider the wave
function and its superpositions as a concrete and physically existing
object.


> You say others think differently, but don't allude to who those other
> thinkers are or what their thoughts are.  Do you have an explanation for
> quantum computers that works with the assumption the wave function is not
> real?
>
>
> Yes. The particular person I was thinking of here is Scott Aaronson. He is
> no fan of Deutsh's approach to quantum computing and many worlds. He points
> out that quantum computers rely on interference 

Re: Is the "bubble multi-verse" and "qm many-worlds" the same thing?

2018-06-18 Thread Bruno Marchal


> On 15 Jun 2018, at 12:33, Telmo Menezes  wrote:
> 
> On 15 June 2018 at 02:55,   wrote:
>> 
>> 
>> On Thursday, June 14, 2018 at 8:15:59 PM UTC, agrays...@gmail.com wrote:
>>> 
>>> 
>>> 
>>> On Wednesday, June 13, 2018 at 11:30:27 PM UTC, Jason wrote:
 
 
 Physical Theories, Eternal Inflation, and Quantum Universe, Yasunori
 Nomura
 
 We conclude that the eternally inflating multiverse and many worlds in
 quantum mechanics are the same. Other important implications include:
 global spacetime
 can be viewed as a derived concept; the multiverse is a transient
 phenomenon during the
 world relaxing into a supersymmetric Minkowski state. We also present a
 theory of “initial
 conditions” for the multiverse. By extrapolating our framework to the
 extreme, we arrive at a
 picture that the entire multiverse is a fluctuation in the stationary,
 fractal “mega-multiverse,”
 in which an infinite sequence of multiverse productions occurs.
 
 "Therefore, we conclude that the multiverse is the same as (or a specific
 manifestation
 of ) many worlds in quantum mechanics."
 
 "In eternal inflation, however, one first picks a causal patch; then one
 looks for observers in it.” Our framework does not follow this approach. We
 instead pick an observer first, and then construct the relevant spacetime
 regions associated with it.
 
 Instead of admitting the existence of the “beginning,” we may require
 that the quantum observer principle is respected for the whole history of
 spacetime. In this case, the beginning of our multiverse is a fluctuation 
 of
 a larger structure, whose beginning is also a fluctuation of an even larger
 structure, and this series goes on forever. This leads to the picture that
 our multiverse arises as a fluctuation in a huge, stationary
 “megamultiverse,” which has a fractal structure."
 
 
 The Multiverse Interpretation of Quantum Mechanics, Raphael Bousso and
 Leonard Susskind
 
 In both the many-worlds interpretation of quantum mechanics and the
 multiverse
 of eternal inflation the world is viewed as an unbounded collection of
 parallel universes.
 A view that has been expressed in the past by both of us is that there is
 no need to
 add an additional layer of parallelism to the multiverse in order to
 interpret quantum
 mechanics. To put it succinctly, the many-worlds and the multiverse are
 the same
 thing [1].
 
 
 Jason
>>> 
>>> 
>>> Not right. Not even wrong. AG.
>> 
>> 
>> Eternal inflation and string theory imply universes created by natural
>> processes. The jury is out on those. OTOH, the MWI has human beings creating
>> universes by going into a lab and doing trivial quantum experiments. Of
>> course they're they same (for idiots). AG
> 
> The MWI does not propose that new universes are created specifically
> by certain experiences in the lab. It proposes that this universe
> branching is a fundamental natural mechanism -- that it happens for
> every quantum-level event that we perceive as random from our branch.
> It's an attempt to describe nature by making sense of experimental
> results, the same way as string theory and other theories.
> 
> It is perhaps the size of the multiverse implied by MWI that makes it
> hard to believe. It is good to be skeptical of our own "common sense"
> on these topics, because human common sense has been wrong many times
> before in the history of science. Consider the size of the visible
> universe, something that is uncontroversial nowadays, but that would
> sound like complete lunacy not so long ago.


My feeling is that here mechanist gives comfort, as there is only 0, 1, 2, 3, 
…, or only K, S, KK, …

A notion of physical world can still make sense, as the least set of 
proposition true in all the description of realities consistent with our 
(indexical) memories”. But its multiplicity is no more astonishing than the 
infinity of universal number relations, or of the prime numbers. 

If QM is correct, and well married with GR, the bubble mutilverse should be 
given by an Everett quantum mechanical description of the vacuum, which itself 
should be a quantum universal dovetailing extractable from the logic of the 
observable of the universal machine. 

What we see is only a tiny part of the physical reality which is itself only 
the border of something much bigger, infinitely bigger, and still put under the 
rug to avoid admitting our ignorance. 
The adults have to feel like they have to kill the questioning kid inside them 
to … mate and get a job ...

Bruno





> 
> Telmo.
> 
>> --
>> You received this message because you are subscribed to the Google Groups
>> "Everything List" group.
>> To unsubscribe from this group and stop receiving emails from it, send an
>> email to everything-list+unsubscr...@googlegroups.com.
>> To post 

Re: Is the "bubble multi-verse" and "qm many-worlds" the same thing?

2018-06-18 Thread Jason Resch
On Mon, Jun 18, 2018 at 5:14 PM,  wrote:

>
>
> On Monday, June 18, 2018 at 8:33:23 PM UTC, Jason wrote:
>>
>>
>>
>> On Mon, Jun 18, 2018 at 2:50 PM,  wrote:
>>
>>>
>>>
>>> On Monday, June 18, 2018 at 1:11:23 PM UTC, Jason wrote:



 On Mon, Jun 18, 2018 at 7:42 AM,  wrote:

>
>
> On Monday, June 18, 2018 at 12:30:02 PM UTC, Jason wrote:
>>
>>
>>
>> On Mon, Jun 18, 2018 at 7:04 AM,  wrote:
>>
>>>

>>> *No "but's" about it! That's the 800 pound gorilla in the room that
>>> Many Worlder's studiously ignore. There's no proposed mechanism to 
>>> explain
>>> the copying! They will appeal to the mathematics, which they rely on
>>> uncritically. But it's obvious that such reliance can lead to ridiculous
>>> results if taken literally. For example, Maxwell's equations have plane
>>> wave solutions, but plane waves don't exist! Think about what a plane 
>>> wave
>>> is; all peaks and troughs extend to infinity, advancing along an 
>>> infinite
>>> plane in every direction. And No, I don't have a classical view of 
>>> reality,
>>> but neither do I fall in love with BS. AG*
>>>



>> Rather thank think of it like copying, think of it like this:
>>
>> - When a photon hits a semi-silvered mirror, it splits and takes two
>> paths.  Now, both the photon's position and its direction are 
>> multi-valued.
>>
>
> *Does it split into two photons, each having the same energy as the
> original photon?*
>

>> It's one photon, with a property that is multi-valued.
>>
>>
>>> * If so, where does the added energy come from.*
>

>> N/A - it's still just one photon
>>
>>
>>> * Or does it split in half?*
>

>> N/A - it's still just one photon
>>
>>
>>>
> * Or does it take two possible paths, not both paths simultaneously?
> TIA, AG*
>

>> If it's velocity property is multi-valued, you could say it follows more
>> than one path at the same time.
>> But this is not necessary the case. The superposition might be in terms
>> of polarization, rather than its velocity, and in that case it's only takes
>> one path.
>>
>> Jason
>>
>
> *So it takes one of two paths, presumably with a 50% probability. How does
> this result in copies, of the photon or possibly entire universes?  Sounds
> like you're grasping at straws to explain the copying mechanism. AG*
>

That's not what I said.  You should re-read my original answer and
description.

Jason



>
>
>>
>>>
 Think of it being only a change to the photon's properties.

>>>
>>> *If you are able, and have the time, please answer my question above,
>>> specifically. TIA, AG*
>>>
>>>
 It's not really two photons, it's a single photon, now holding multiple
 values for some of its properties (which include velocity, position,
 frequency, polarization, etc.)  After interacting with a semi-silvered
 mirror, the same thing always happens to a photon, it takes on multiple
 values for its velocity.

 Jason



>
>
>> - We can use full-silvered mirrors to redirect this "multi-valued"
>> photon such that both of its positions will converge (while nonetheless
>> traveling in different directions on that intercept course)
>> - We can place an electron at that point of interception, such that
>> this multi-velocity photon interacts with and strikes this electron 
>> (which
>> right now only has one position and zero velocity)
>> - When the multi-velocity photon hits the electron, the electron now
>> has two velocities (you could view it as in one case, it was hit from
>> below, and in another case it was hit from the side).
>>
>> So we see a clear case, where a photon, being in a super-position of
>> states, when it interacts with an electron, it's multiple states transfer
>> to that electron such that it now ends up in a super position, and will
>> have different positions and velocities over time.  Anything that 
>> interacts
>> with that electron, or doesn't interact with it will also become
>> multi-valued.
>>
>> Let's say there is a detection screen to the right of the electron,
>>  the multi-valued electron interacts with the atoms in the detection
>> screen. Because the electron's position is multi-valued, and only one of
>> the electron's velocity's intercept the screen, the atoms in the 
>> detection
>> screen also enter a super position, of having interacted with the 
>> electron
>> and having not interacted with the electron (the upward moving electron
>> velocity misses the screen).  So now there is a superposition of states 
>> in
>> the room where a flash flash of light from the screen has occurred, and
>> where no flash of light on the screen has occurred.
>>
>> You can continue this example 

Re: Is the "bubble multi-verse" and "qm many-worlds" the same thing?

2018-06-18 Thread agrayson2000


On Monday, June 18, 2018 at 3:23:12 PM UTC, John Clark wrote:
>
> On Sun, Jun 17, 2018 at 9:54 PM, > 
> wrote:
>
> >> What about it, what is your theory of decoherence and how does it make 
>>> the CI less dumb?  
>>
>>  
>> > *Not that I'm a great fan of decoherence theory, but it doest includes 
>> the apparatus, observer, and the rest of the environment in the measuring 
>> process.*
>
>
> I know what quantum decoherence is but I've never heard of the term 
> "decoherence theory" before and I don't see how anyone can talk about 
> decoherence without referring to the rest of the environment. 
>

*This subject has been discussed on this MB with reference to the 
environment. Do you really expect me to rehash all the pros and cons of the 
theory or interpretation before I can reference it in any way?  Odd that 
you complain about no reference to the environment when I did just THAT in 
my comment you are responding to. AG*
 

> If X is the only thing in the universe or the only thing that’s important 
> then there is nothing X can become cohered or de-cohered from. 
>
> *>>>  I have always regarded Bell results as paradoxical, or if you prefer 
 unintelligible,*
>>>
>>>  
>
> >>It's not paradoxical because its not self contradictory and its not 
>>> unintelligible because the results are clear as a Bell (pun intended), they 
>>> are just very very odd.  
>>
>>
>> > *Do us all a favor and stop playing word games. *
>
>
> Now you sound like Bruno. Do you really thing you can give words vague 
> ephemeral meanings and still do science?
>

*Well, Bruno is right at least about one thing; you're a bully. In this 
context, you insist on splitting hairs about specific definitions of words, 
when you know what I mean. AG *

>
>
> *>What you call "odd", can easily been seen as paradoxical or 
>> unintelligible *
>
>
> I don’t think we’re ever going to be able to figure out anything if the 
> meanings of words change at our whim from day to day. The reductio ad 
> absurdum proof is actually poorly named because it is not good enough to 
> show that a proposition will lead to something odd or even very odd to 
> prove that the proposition must be untrue, you’ve got to show it is 
> paradoxical, that is to say self contradictory. If Many World’s or the 
> Transactional Interpretation was true it would means some very odd things 
> were going on behind the scenes, but that doesn’t prove either of them is 
> untrue. In fact now that the Bell experiments have been performed one thing 
> we know for sure is that some very odd things ARE going on behind the 
> scenes, we’re just not sure exactly what they are. So if your explanation 
> is not very very odd it can’t be right.
>

*I'm not saying the explanation of copying can't be odd if it exists; 
rather, to make the case for the MWI, it's up to the advocates to offer 
some plausible proposals about the mechanism. But so far, in years of 
discussing this issue with advocates, I've never seen any proposal that has 
any teeth, or indeed any proposal whatsoever. AG*

I don’t know what it would mean if Copenhagen turned out to be true because 
even those who say they believe in it can’t agree among themselves what 
exactly the Copenhagen Interpretation is saying other than “shut up and 
calculate”.
 

*The advocates of the MWI more or less do the same. They're all over the 
map about the copying issue, role of branches, etc. AG *

>
>
> *> insofar as it can't be understood in terms of how we perceive space, or 
>> spatial extent, and of course causality *
>
>
> So to have any hope of understanding what’s going on we’re going to have 
> to abandon comfortable concepts like causality and the normal way we 
> perceive space and embrace something much stranger. 
>
> >> Are you confusing plane waves with advanced waves?
>>
>>
>> >*Definitely not. Plane waves don't exist except possibly in your 
>> imagination.  AG*
>
>
> To a good approximation a Laser produces plane waves, the electromagnetic 
> wave fronts form parallel planes, that’s why a Laser beam spreads out very 
> little with distance unlike a light bulb which produces a spherical wave. 
> But I don’t see what this has to do with quantum interpretation and I don’t 
> know of any physicists who thinks plane waves are more controversial than 
> spherical waves.
>

*Spherical waves exist. They start of out with a zero radius, which 
increases with time. Plane waves start out with infinite extent in every 
direction, so they don't exist. This isn't an issue for physicists since 
they haven't considered the subtle gap between mathematics and reality. I 
offered the example just to point out the fallacy or error in excessive 
reliance on mathematics, as you do wrt the Schroedinger equation and what 
it implies. AG *

>

-- 
You received this message because you are subscribed to the Google Groups 
"Everything List" group.
To unsubscribe from this group and stop receiving emails from it, send an email 
to 

Re: Is the "bubble multi-verse" and "qm many-worlds" the same thing?

2018-06-18 Thread Russell Standish
On Mon, Jun 18, 2018 at 06:18:47AM -0500, Jason Resch wrote:
> On Sun, Jun 17, 2018 at 6:37 PM, Russell Standish 
> wrote:
> 
> > I've been meaning to get this in publishable form, but time and other
> > commitments have gotten in my way.
> >
> >
> I (and I know many others on this list and elsewhere) eagerly await and
> look forward this.
> 

Well it's good in the sense that business is going well. But not so
good in that my fundamental research program has gone on the
backburner. With a bit of luck, one of my projects will go so well
that I'll be able to "retire" to doing full time research... Dream on.

> > In the meantime, another problem came to my attention from Markus
> > Mueller (arxiv:1712.0181), where he points out that it is an open
> > question whether transition probability for process on strings is
> > naturally Markovian. The latter portion of my proof, in particular
> > (D.13) is assuming a Markovian process.
> >
> >
> 
> Thanks for the reference, I will check it out.
> 
> Might combining your theory with a theory of computation (like the UDA) be
> helpful in linking or otherwise tying together successively observed bit
> strings?

Well I have had a lot of time to think about it (couple of decades),
and the link has kind of stumped me to date. However Markus Mueller's
work, of which I have only recently become aware, might be the missing
link. His setup is based around a transition graph on bitstrings. The
bitstrings most likely correspond to observer moment subsets of
everything in my theory, but the graph probably corresponds to the
Kripke network that arises in one of Bruno's hypostases.

But it'll probably require a younger mind than mine. But most younger
minds will not be financially or professionally secure enough to
tackle what has a whiff of crank science about it, sadly.


> 
> I found the book "Trespassing on Einstein's Lawn
> "
> to be quite remarkable in breaking down the laws of physics to being the
> bare minimum that is necessary to ensure consistency between observers. It
> might be a fruitful avenue to explore, as it seems at least possibly
> related to your effort.
>

Thanks for the suggestion. I've always enjoyed her columns in New
Scientist. I might wait for a Kindle version though, as getting that
book in physical for to Oz usually doubles or triples the
price. Actually, I noticed that Carlo Rovelli has a book out that
might be worth a read too.


-- 


Dr Russell StandishPhone 0425 253119 (mobile)
Principal, High Performance Coders
Visiting Senior Research Fellowhpco...@hpcoders.com.au
Economics, Kingston University http://www.hpcoders.com.au


-- 
You received this message because you are subscribed to the Google Groups 
"Everything List" group.
To unsubscribe from this group and stop receiving emails from it, send an email 
to everything-list+unsubscr...@googlegroups.com.
To post to this group, send email to everything-list@googlegroups.com.
Visit this group at https://groups.google.com/group/everything-list.
For more options, visit https://groups.google.com/d/optout.


Re: Is the "bubble multi-verse" and "qm many-worlds" the same thing?

2018-06-18 Thread Bruce Kellett

From: *Jason Resch* mailto:jasonre...@gmail.com>



On Mon, Jun 18, 2018 at 7:38 AM, Bruce Kellett 
mailto:bhkell...@optusnet.com.au>> wrote:


From: *Jason Resch* mailto:jasonre...@gmail.com>>



In the EPR experiment, a pair of photons is created.  Each photon
is in a super position of every possible polarization, and
because it is created as a pair, it's dual in the superposed
state always has exactly the opposite polarization (rotated 180
degrees).



OK.


When you perform a measurement of your left-traveling photon on
Earth, you become entangled (correlated) with it, and all the
possible states of that photon, when measured, leak into the
room, starting with the measuring device, then your eyes, then
your brain, then your notebook, etc. until now everything is in
the room, and soon Earth is now in many states which contagiously
spread from that photon.


OK. Your result (and you) become entangled with your environment.


Also, because the photon you measured was entangled (correlated)
with its pair in the superposition, whatever result you measure
for the photon's polarization tells you immediately what the
polarization of its pair is (in your branch at least).  So any
future communication you get from me on Pluto will necessarily
align with the result you measured.


This is where the mistake creeps in. My measurement tells me the
polarization of the entangled photon in the branch in which my
measurement was made. When you come to measure your entangled
photon on Pluto, how do you know what branch my measurement was
made in? You are at a spacelike separation from me, and completely
independent. So I ask again, how come you assume that your
measurement will be in the same branch as mine was?


Let's make it more concrete and say there are only 360 possible 
polarizations, each having an equal probability.


That is not a very good way to look at it. The photon is not in a 
superposition of all possible polarization states. You cannot write the 
photon wave function as such a superposition:


 |psi> = Sum_i a_i |i> for i running over all 360 possibilities in 
the case you outline.


The most you can ever do is write the state as a superposition of the 
two possible polarizations in any particular direction. Thus:


   |psi> = (|+> + |->), ignoring normalization factors.

This can be written for |+> and |-> being the polarization eigenstates 
in any chosen direction. But not all directions at once.


The photon pair is then in a superposition of 360 possible states.  
The photon pair must be considered as a single object, because if your 
photon is 240 degrees, mine is -240 (120 degrees), and so on. There 
are only 360 possible values that could be obtained from measurement, 
not (360 * 360).


There are only ever two possible polarization states, although these can 
be defined in any of an infinity of possible directions. But once a 
basis is chosen, that defines the total superposition.


When I measure my photon on Pluto, I am self-locating myself to a 
branch (one of 360 possible branches of the wave function 
corresponding to each of the 360 possible polarization of the photon 
on Pluto).  Once I have located myself to this branch, I may not know 
which measurement angle you will set your filter at, I remain in a 
super position of all possible measurement angles you might choose 
(let's say there are 3 possible measurement angles).


After your measurement, you first transmit, not your result, but your 
measurement angle.  Once the photons from this radio signal reach me, 
I have located myself to one of the 3 possibilities for the 
measurement angle. At this moment, I have all the information I need 
to be able to completely predict the statistics of your measurement 
result, based on my measurement result and angle which I knew since 
the time of measurement, and now with your measurement angle 
information having reached me.  When you transmit your measurement 
result to me, I find it in agreement with my expectations for having 
located myself to a branch that had (360 * 3) possibilities that were 
unknown to me at the time prior to performing the experiment.


We can make it simpler than this. Even though the two measurements are 
supposed to be independent -- at independent angles -- we can relax this 
for the purposes of illustration and say that the two experimenters 
agree to both measure at some particular angle. If you on Pluto make a 
measurement at this angle, there are only two possible outcomes, the |+> 
or |-> states in the notation I have used above. So you are in a 
superposition of two possible worlds, one for each result. Because of 
conservation of angular momentum, you know that if you got the |+> 
result, then the other photon of the entangled pair would, if measured 
at the same angle, give |->, and similarly if you are in the branch that 
got a |-> result.

Re: Is the "bubble multi-verse" and "qm many-worlds" the same thing?

2018-06-18 Thread agrayson2000


On Monday, June 18, 2018 at 9:55:55 PM UTC, Jason wrote:
>
>
>
> On Mon, Jun 18, 2018 at 5:14 PM, > 
> wrote:
>
>>
>>
>> On Monday, June 18, 2018 at 8:33:23 PM UTC, Jason wrote:
>>>
>>>
>>>
>>> On Mon, Jun 18, 2018 at 2:50 PM,  wrote:
>>>


 On Monday, June 18, 2018 at 1:11:23 PM UTC, Jason wrote:
>
>
>
> On Mon, Jun 18, 2018 at 7:42 AM,  wrote:
>
>>
>>
>> On Monday, June 18, 2018 at 12:30:02 PM UTC, Jason wrote:
>>>
>>>
>>>
>>> On Mon, Jun 18, 2018 at 7:04 AM,  wrote:
>>>

>
 *No "but's" about it! That's the 800 pound gorilla in the room that 
 Many Worlder's studiously ignore. There's no proposed mechanism to 
 explain 
 the copying! They will appeal to the mathematics, which they rely on 
 uncritically. But it's obvious that such reliance can lead to 
 ridiculous 
 results if taken literally. For example, Maxwell's equations have 
 plane 
 wave solutions, but plane waves don't exist! Think about what a plane 
 wave 
 is; all peaks and troughs extend to infinity, advancing along an 
 infinite 
 plane in every direction. And No, I don't have a classical view of 
 reality, 
 but neither do I fall in love with BS. AG*

>
>
>
>>> Rather thank think of it like copying, think of it like this:
>>>
>>> - When a photon hits a semi-silvered mirror, it splits and takes two 
>>> paths.  Now, both the photon's position and its direction are 
>>> multi-valued.
>>>
>>
>> *Does it split into two photons, each having the same energy as the 
>> original photon?*
>>
>
>>> It's one photon, with a property that is multi-valued.
>>>  
>>>
 * If so, where does the added energy come from.*
>>
>
>>> N/A - it's still just one photon
>>>  
>>>
 * Or does it split in half?*
>>
>
>>> N/A - it's still just one photon
>>>  
>>>

>> * Or does it take two possible paths, not both paths simultaneously? 
>> TIA, AG*
>>
>
>>> If it's velocity property is multi-valued, you could say it follows more 
>>> than one path at the same time.
>>> But this is not necessary the case. The superposition might be in terms 
>>> of polarization, rather than its velocity, and in that case it's only takes 
>>> one path.
>>>  
>>> Jason
>>>
>>
>> *So it takes one of two paths, presumably with a 50% probability. How 
>> does this result in copies, of the photon or possibly entire universes?  
>> Sounds like you're grasping at straws to explain the copying mechanism. AG*
>>
>
> That's not what I said.  You should re-read my original answer and 
> description.
>
> Jason
>


*I am asking for a simple answer to a simple question. If you do the 
experiment one photon at a time, what is observed? What you have offered is 
an interpretation of what happens. AG *

>
>  
>
>>
>>  
>>>

> Think of it being only a change to the photon's properties. 
>

 *If you are able, and have the time, please answer my question above, 
 specifically. TIA, AG*
  

> It's not really two photons, it's a single photon, now holding 
> multiple values for some of its properties (which include velocity, 
> position, frequency, polarization, etc.)  After interacting with a 
> semi-silvered mirror, the same thing always happens to a photon, it takes 
> on multiple values for its velocity.
>
> Jason
>
>  
>
>>  
>>
>>> - We can use full-silvered mirrors to redirect this "multi-valued" 
>>> photon such that both of its positions will converge (while nonetheless 
>>> traveling in different directions on that intercept course)
>>> - We can place an electron at that point of interception, such that 
>>> this multi-velocity photon interacts with and strikes this electron 
>>> (which 
>>> right now only has one position and zero velocity)
>>> - When the multi-velocity photon hits the electron, the electron now 
>>> has two velocities (you could view it as in one case, it was hit from 
>>> below, and in another case it was hit from the side).
>>>
>>> So we see a clear case, where a photon, being in a super-position of 
>>> states, when it interacts with an electron, it's multiple states 
>>> transfer 
>>> to that electron such that it now ends up in a super position, and will 
>>> have different positions and velocities over time.  Anything that 
>>> interacts 
>>> with that electron, or doesn't interact with it will also become 
>>> multi-valued.
>>>
>>> Let's say there is a detection screen to the right of the electron, 
>>>  the multi-valued electron interacts with the atoms in the detection 
>>> screen. Because the electron's position is multi-valued, and only one 
>>> of 
>>> the electron's velocity's 

Re: Do we live within a Diophantine equation?

2018-06-18 Thread smitra

On 17-06-2018 22:42, Jason Resch wrote:

Hi Lawrence,

Is the evolution of states of the wave function computable? If so then
the result of MRDP implies it is Diophantine.

Jason


Or you could try to see if QM could be a meta-theory that arises when 
you try to give a statistical description of the set of all these 
Diophantine sets. I tried to do something similar with the set of 
algorithms a few years ago, getting a half-baked result, some hints at 
how quantum field theory could arise from this.


You want to compute the probability that an observer that's encoded by 
some mathematical structure has some given information content. So, if 
you observe the outcome of an experiment, that's information in your 
brain. But your brain is supposed to be some mathematical structure and 
that then contains also that specific information about the outcome of 
the experiment. Probabilities should presumably be obtained by counting 
the number of states compatible with some observation, but we must then 
impose the restriction that we're only going to count states that 
correspond to some given observer making that observation. If observers 
are specified algorithms that are specified by a set of input and 
corresponding output states, then we must sum over all input and output 
states, that fit each other. This is mathematically inconvenient, one 
can replace such a summation by an unrestricted summation by including 
Kronecker delta factors:


delta_{r,s} = 0 if r is not equal to s, otherwise it is 1.

One can then write:

delta_{r,s} = Integral from 0 to 1 of Exp[2 pi i (r-s) theta] dtheta

One can then sum over the variables freely, but one is then left with 
integrations over many different theta variables. The idea is then that 
in the limit of a large number of variables you can work with coarse 
grained averages over the theta variables, you end up with something 
similar to the path integral formulation of QFT.


Saibal


--
You received this message because you are subscribed to the Google Groups 
"Everything List" group.
To unsubscribe from this group and stop receiving emails from it, send an email 
to everything-list+unsubscr...@googlegroups.com.
To post to this group, send email to everything-list@googlegroups.com.
Visit this group at https://groups.google.com/group/everything-list.
For more options, visit https://groups.google.com/d/optout.


Re: Is the "bubble multi-verse" and "qm many-worlds" the same thing?

2018-06-18 Thread John Clark
On Mon, Jun 18, 2018 at 8:37 PM,  wrote:

> Oh, so now the expansion of the universe is effecting photon energy in a
> quantum experiment?


If space is expanding thee is no way a photons energy could not be reduced.
Where did that energy go? It didn't have to go anywhere because energy is
not conserved cosmically.


> *> Soon you'll be claiming the Milky Way is coming apart due to the
> expansion. AG*


​That could be, it depends on if the acceleration of the universe is itself
accelerating. Its called The Big Rip. From

​https://en.wikipedia.org/wiki/Big_Rip


​"*​*
*galaxies would first be separated from each other. About 60 million years
before the Big Rip, gravity would be too weak to hold the Milky Way and
other individual galaxies together. Galaxies would be destroyed as stars
separate from the main black hole. Approximately three months before the
Big Rip, the Solar System (or systems similar to our own at this time, as
the fate of the Solar System 22 billion years in the future is
questionable) would be gravitationally unbound. Planets would be detached
from the star’s orbit. In the last minutes, stars and planets would be torn
apart, and an extremely short amount of time before the Big Rip, atoms
would be destroyed. At the time the Big Rip occurs, even spacetime itself
will be ripped apart*
​"​

​John K Clark​

-- 
You received this message because you are subscribed to the Google Groups 
"Everything List" group.
To unsubscribe from this group and stop receiving emails from it, send an email 
to everything-list+unsubscr...@googlegroups.com.
To post to this group, send email to everything-list@googlegroups.com.
Visit this group at https://groups.google.com/group/everything-list.
For more options, visit https://groups.google.com/d/optout.


Re: Is the "bubble multi-verse" and "qm many-worlds" the same thing?

2018-06-18 Thread John Clark
On Mon, Jun 18, 2018 at 2:50 PM,  wrote:

*​> ​Does it split into two photons, each having the same energy as the
> original photon?​ * If so, where does the added energy come from.


It doesn't need to come from anywhere because we've known for nearly a
century that in General Relativity energy is NOT conserved at the largest
scale. Consider light energy, as the universe expands all the light photons
in it gets red-shifted and lose energy. Or consider the energy of empty
space, Dark Energy. As the universe expands there is more space and thus
more Dark Energy.

Another way of looking at it is with Noether's theorem, it says energy is
conserved if the laws that govern the way particles move does not change
with time, but they do change with time, the space through which particles
move is not only expanding it is accelerating. So the conservation of
energy is approximately true locally but not cosmically.

John K Clark

-- 
You received this message because you are subscribed to the Google Groups 
"Everything List" group.
To unsubscribe from this group and stop receiving emails from it, send an email 
to everything-list+unsubscr...@googlegroups.com.
To post to this group, send email to everything-list@googlegroups.com.
Visit this group at https://groups.google.com/group/everything-list.
For more options, visit https://groups.google.com/d/optout.


Re: Is the "bubble multi-verse" and "qm many-worlds" the same thing?

2018-06-18 Thread agrayson2000


On Monday, June 18, 2018 at 2:40:20 PM UTC, Bruno Marchal wrote:
>
>
> > On 15 Jun 2018, at 12:33, Telmo Menezes  > wrote: 
> > 
> > On 15 June 2018 at 02:55,  > wrote: 
> >> 
> >> 
> >> On Thursday, June 14, 2018 at 8:15:59 PM UTC, agrays...@gmail.com 
> wrote: 
> >>> 
> >>> 
> >>> 
> >>> On Wednesday, June 13, 2018 at 11:30:27 PM UTC, Jason wrote: 
>  
>  
>  Physical Theories, Eternal Inflation, and Quantum Universe, Yasunori 
>  Nomura 
>  
>  We conclude that the eternally inflating multiverse and many worlds 
> in 
>  quantum mechanics are the same. Other important implications include: 
>  global spacetime 
>  can be viewed as a derived concept; the multiverse is a transient 
>  phenomenon during the 
>  world relaxing into a supersymmetric Minkowski state. We also present 
> a 
>  theory of “initial 
>  conditions” for the multiverse. By extrapolating our framework to the 
>  extreme, we arrive at a 
>  picture that the entire multiverse is a fluctuation in the 
> stationary, 
>  fractal “mega-multiverse,” 
>  in which an infinite sequence of multiverse productions occurs. 
>  
>  "Therefore, we conclude that the multiverse is the same as (or a 
> specific 
>  manifestation 
>  of ) many worlds in quantum mechanics." 
>  
>  "In eternal inflation, however, one first picks a causal patch; then 
> one 
>  looks for observers in it.” Our framework does not follow this 
> approach. We 
>  instead pick an observer first, and then construct the relevant 
> spacetime 
>  regions associated with it. 
>  
>  Instead of admitting the existence of the “beginning,” we may require 
>  that the quantum observer principle is respected for the whole 
> history of 
>  spacetime. In this case, the beginning of our multiverse is a 
> fluctuation of 
>  a larger structure, whose beginning is also a fluctuation of an even 
> larger 
>  structure, and this series goes on forever. This leads to the picture 
> that 
>  our multiverse arises as a fluctuation in a huge, stationary 
>  “megamultiverse,” which has a fractal structure." 
>  
>  
>  The Multiverse Interpretation of Quantum Mechanics, Raphael Bousso 
> and 
>  Leonard Susskind 
>  
>  In both the many-worlds interpretation of quantum mechanics and the 
>  multiverse 
>  of eternal inflation the world is viewed as an unbounded collection 
> of 
>  parallel universes. 
>  A view that has been expressed in the past by both of us is that 
> there is 
>  no need to 
>  add an additional layer of parallelism to the multiverse in order to 
>  interpret quantum 
>  mechanics. To put it succinctly, the many-worlds and the multiverse 
> are 
>  the same 
>  thing [1]. 
>  
>  
>  Jason 
> >>> 
> >>> 
> >>> Not right. Not even wrong. AG. 
> >> 
> >> 
> >> Eternal inflation and string theory imply universes created by natural 
> >> processes. The jury is out on those. OTOH, the MWI has human beings 
> creating 
> >> universes by going into a lab and doing trivial quantum experiments. Of 
> >> course they're they same (for idiots). AG 
> > 
> > The MWI does not propose that new universes are created specifically 
> > by certain experiences in the lab. It proposes that this universe 
> > branching is a fundamental natural mechanism -- that it happens for 
> > every quantum-level event that we perceive as random from our branch. 
> > It's an attempt to describe nature by making sense of experimental 
> > results, the same way as string theory and other theories. 
> > 
> > It is perhaps the size of the multiverse implied by MWI that makes it 
> > hard to believe. It is good to be skeptical of our own "common sense" 
> > on these topics, because human common sense has been wrong many times 
> > before in the history of science. Consider the size of the visible 
> > universe, something that is uncontroversial nowadays, but that would 
> > sound like complete lunacy not so long ago. 
>
>
> My feeling is that here mechanist gives comfort, as there is only 0, 1, 2, 
> 3, …, or only K, S, KK, … 
>
> A notion of physical world can still make sense, as the least set of 
> proposition true in all the description of realities consistent with our 
> (indexical) memories”. But its multiplicity is no more astonishing than the 
> infinity of universal number relations, or of the prime numbers. 
>
> If QM is correct, and well married with GR, the bubble mutilverse should 
> be given by an Everett quantum mechanical description of the vacuum, which 
> itself should be a quantum universal dovetailing extractable from the logic 
> of the observable of the universal machine. 
>
> What we see is only a tiny part of the physical reality which is itself 
> only the border of something much bigger, infinitely bigger, and still put 
> under the rug to avoid admitting our ignorance. 
>

*If 

Re: Is the "bubble multi-verse" and "qm many-worlds" the same thing?

2018-06-18 Thread agrayson2000


On Monday, June 18, 2018 at 11:46:15 PM UTC, John Clark wrote:
>
> On Mon, Jun 18, 2018 at 2:50 PM, > 
> wrote:
>
> *​> ​Does it split into two photons, each having the same energy as the 
>> original photon?​ * If so, where does the added energy come from.
>
>
> It doesn't need to come from anywhere because we've known for nearly a 
> century that in General Relativity energy is NOT conserved at the largest 
> scale. Consider light energy, as the universe expands all the light photons 
> in it gets red-shifted and lose energy. Or consider the energy of empty 
> space, Dark Energy. As the universe expands there is more space and thus 
> more Dark Energy.
>


*Oh, so now the expansion of the universe is effecting photon energy in a 
quantum experiment? Soon you'll be claiming the Milky Way is coming apart 
due to the expansion. AG*

>
> Another way of looking at it is with Noether's theorem, it says energy is 
> conserved if the laws that govern the way particles move does not change 
> with time, but they do change with time, the space through which particles 
> move is not only expanding it is accelerating. So the conservation of 
> energy is approximately true locally but not cosmically. 
>
> John K Clark
>

-- 
You received this message because you are subscribed to the Google Groups 
"Everything List" group.
To unsubscribe from this group and stop receiving emails from it, send an email 
to everything-list+unsubscr...@googlegroups.com.
To post to this group, send email to everything-list@googlegroups.com.
Visit this group at https://groups.google.com/group/everything-list.
For more options, visit https://groups.google.com/d/optout.


Re: Is the "bubble multi-verse" and "qm many-worlds" the same thing?

2018-06-18 Thread John Clark
On Mon, Jun 18, 2018 at 7:54 PM,  wrote:

*> If you accept Inflation, the universe is many orders of magnitude larger
> than what we can observe. How much larger depends on the model of Inflation
> one applies. However, AFAIK, there's no persuasive theory that claims its
> extent in space or time is INFINITE.*


During inflation the expansion of the universe was exponential which means
it had a fixed doubling time, in this case every 10^-37 seconds the
diameter of the universe doubled. In 10^-35 seconds it doubled a hundred
times and it probably continued doubling for much longer than 10^-35
seconds.But why did it ever end?

According to Andrei Linde inflation never did end. Alan Guth, the inventor
of inflation, postulated an inflation field that decayed away in a process
somewhat analogous to radioactive half life, and after the decay the
universe expanded at a much much much more leisurely pace. But then Linde
proved that for Guth's idea to work the inflation field had to expand
faster than it decayed, Linde called it "Eternal Inflation". Linde showed
that for every volume in which the inflation field decays away 2 other
volumes don't decay. So one universe becomes 3, the field decays in one
universe but not in the other 2, then both of those two universes splits in
3 again and the inflation field decays away in two of them but doesn't
decay in the other 4.  And it goes on like this forever creating a
multiverse.

*​>​I haven't delved deeply into this issue. I tend to the position that
> the universe MIGHT be infinite in space and time, but NOT our local bubble,
> which I believe is finite. AG*


Our observable universe is not only finite its getting smaller due to the
accelerating expansion of space. In a trillion years or so it will consist
of Milkdromeda (the combined Milky Way and Andromeda galaxy) and that's it
because everything else will be moving away from us faster than light and
thus be unobservable
​.

John K Clark​

-- 
You received this message because you are subscribed to the Google Groups 
"Everything List" group.
To unsubscribe from this group and stop receiving emails from it, send an email 
to everything-list+unsubscr...@googlegroups.com.
To post to this group, send email to everything-list@googlegroups.com.
Visit this group at https://groups.google.com/group/everything-list.
For more options, visit https://groups.google.com/d/optout.


Re: Is the "bubble multi-verse" and "qm many-worlds" the same thing?

2018-06-18 Thread agrayson2000


On Tuesday, June 19, 2018 at 12:53:29 AM UTC, John Clark wrote:
>
>
> On Mon, Jun 18, 2018 at 7:54 PM, > 
> wrote:
>
> *> If you accept Inflation, the universe is many orders of magnitude 
>> larger than what we can observe. How much larger depends on the model of 
>> Inflation one applies. However, AFAIK, there's no persuasive theory that 
>> claims its extent in space or time is INFINITE.*
>
>
> During inflation the expansion of the universe was exponential which means 
> it had a fixed doubling time, in this case every 10^-37 seconds the 
> diameter of the universe doubled. In 10^-35 seconds it doubled a hundred 
> times and it probably continued doubling for much longer than 10^-35 
> seconds.But why did it ever end?
>
> According to Andrei Linde inflation never did end. Alan Guth, the inventor 
> of inflation, postulated an inflation field that decayed away in a process 
> somewhat analogous to radioactive half life, and after the decay the 
> universe expanded at a much much much more leisurely pace. But then Linde 
> proved that for Guth's idea to work the inflation field had to expand 
> faster than it decayed, Linde called it "Eternal Inflation". Linde showed 
> that for every volume in which the inflation field decays away 2 other 
> volumes don't decay. So one universe becomes 3, the field decays in one 
> universe but not in the other 2, then both of those two universes splits in 
> 3 again and the inflation field decays away in two of them but doesn't 
> decay in the other 4.  And it goes on like this forever creating a 
> multiverse.
>
> *​>​I haven't delved deeply into this issue. I tend to the position that 
>> the universe MIGHT be infinite in space and time, but NOT our local bubble, 
>> which I believe is finite. AG*
>
>
> Our observable universe is not only finite its getting smaller due to the 
> accelerating expansion of space. In a trillion years or so it will consist 
> of Milkdromeda (the combined Milky Way and Andromeda galaxy) and that's it 
> because everything else will be moving away from us faster than light and 
> thus be unobservable
> ​.
>

*When I referred to "our local bubble", I meant our entire universe, not 
just the observable region which is shrinking. Even with Eternal Inflation, 
our local bubble remains finite if it began at some TIME in the past; that 
is, if its age if finite. For it to be infinite, I would think it couldn't 
have a beginning. AG *

>
> John K Clark​
>
>
>  
>

-- 
You received this message because you are subscribed to the Google Groups 
"Everything List" group.
To unsubscribe from this group and stop receiving emails from it, send an email 
to everything-list+unsubscr...@googlegroups.com.
To post to this group, send email to everything-list@googlegroups.com.
Visit this group at https://groups.google.com/group/everything-list.
For more options, visit https://groups.google.com/d/optout.


Re: Is the "bubble multi-verse" and "qm many-worlds" the same thing?

2018-06-18 Thread agrayson2000


On Tuesday, June 19, 2018 at 1:10:06 AM UTC, John Clark wrote:
>
>
>
> On Mon, Jun 18, 2018 at 8:37 PM, > 
> wrote:
>
> > Oh, so now the expansion of the universe is effecting photon energy in a 
>> quantum experiment?
>
>
> If space is expanding thee is no way a photons energy could not be 
> reduced. Where did that energy go? It didn't have to go anywhere because 
> energy is not conserved cosmically.
>

*Do us all a favor and stop playing games. In the elapsed time of any 
quantum experiment, or even much, much longer, the expansion rate of the 
universe in the neighborhood of the lab is astronomically tiny and doesn't 
effect the energy of a photon to any measureable degree. AG*

>  
>
>> *> Soon you'll be claiming the Milky Way is coming apart due to the 
>> expansion. AG*
>
>
> ​That could be, it depends on if the acceleration of the universe is 
> itself accelerating. Its called The Big Rip. From 
>

*I meant coming apart NOW. Bringing up the expansion rate of the universe 
when the discussion is about the behavior of a photon in quantum experiment 
is just plain dumb. AG* 

>
> ​https://en.wikipedia.org/wiki/Big_Rip
>
>
> ​"*​*
> *galaxies would first be separated from each other. About 60 million years 
> before the Big Rip, gravity would be too weak to hold the Milky Way and 
> other individual galaxies together. Galaxies would be destroyed as stars 
> separate from the main black hole. Approximately three months before the 
> Big Rip, the Solar System (or systems similar to our own at this time, as 
> the fate of the Solar System 22 billion years in the future is 
> questionable) would be gravitationally unbound. Planets would be detached 
> from the star’s orbit. In the last minutes, stars and planets would be torn 
> apart, and an extremely short amount of time before the Big Rip, atoms 
> would be destroyed. At the time the Big Rip occurs, even spacetime itself 
> will be ripped apart*
> ​"​
>
> ​John K Clark​
>
>
>
>  
>  
>
>

-- 
You received this message because you are subscribed to the Google Groups 
"Everything List" group.
To unsubscribe from this group and stop receiving emails from it, send an email 
to everything-list+unsubscr...@googlegroups.com.
To post to this group, send email to everything-list@googlegroups.com.
Visit this group at https://groups.google.com/group/everything-list.
For more options, visit https://groups.google.com/d/optout.


Re: Is the "bubble multi-verse" and "qm many-worlds" the same thing?

2018-06-18 Thread Jason Resch
On Mon, Jun 18, 2018 at 7:26 PM, Bruce Kellett 
wrote:

> From: Jason Resch < jasonre...@gmail.com
>
>
>
> On Mon, Jun 18, 2018 at 7:38 AM, Bruce Kellett <
> bhkell...@optusnet.com.au> wrote:
>
>> From: Jason Resch 
>>
>>
>>
>> In the EPR experiment, a pair of photons is created.  Each photon is in a
>> super position of every possible polarization, and because it is created as
>> a pair, it's dual in the superposed state always has exactly the opposite
>> polarization (rotated 180 degrees).
>>
>>
>> OK.
>>
>> When you perform a measurement of your left-traveling photon on Earth,
>> you become entangled (correlated) with it, and all the possible states of
>> that photon, when measured, leak into the room, starting with the measuring
>> device, then your eyes, then your brain, then your notebook, etc. until now
>> everything is in the room, and soon Earth is now in many states which
>> contagiously spread from that photon.
>>
>>
>> OK. Your result (and you) become entangled with your environment.
>>
>> Also, because the photon you measured was entangled (correlated) with its
>> pair in the superposition, whatever result you measure for the photon's
>> polarization tells you immediately what the polarization of its pair is (in
>> your branch at least).  So any future communication you get from me on
>> Pluto will necessarily align with the result you measured.
>>
>>
>> This is where the mistake creeps in. My measurement tells me the
>> polarization of the entangled photon in the branch in which my measurement
>> was made. When you come to measure your entangled photon on Pluto, how do
>> you know what branch my measurement was made in? You are at a spacelike
>> separation from me, and completely independent. So I ask again, how come
>> you assume that your measurement will be in the same branch as mine was?
>>
>
> Let's make it more concrete and say there are only 360 possible
> polarizations, each having an equal probability.
>
>
> That is not a very good way to look at it. The photon is not in a
> superposition of all possible polarization states. You cannot write the
> photon wave function as such a superposition:
>
>  |psi> = Sum_i a_i |i> for i running over all 360 possibilities in the
> case you outline.
>
> The most you can ever do is write the state as a superposition of the two
> possible polarizations in any particular direction. Thus:
>
>|psi> = (|+> + |->), ignoring normalization factors.
>
> This can be written for |+> and |-> being the polarization eigenstates in
> any chosen direction. But not all directions at once.
>


I see.

Could you explain the point of error in the following paper?  I've
excerpted the relevant sections if it helps your search.

From: https://arxiv.org/pdf/0902.3827.pdf

*According to quantum mechanics, whichever measurement is performed first
collapses the entangled twin state superposition to a single polarization
state that is identical for both photons.*

*[...]*


*If we wish to know what the probability is of getting the same measurement
for photon 1, we need only figure out what the probability is for a photon
with polarization along θ2 to pass through a filter oriented along θ1. This
probability is easily calculated according to simple trigonometry. Any
arbitrary linear polarization can be thought of as a superposition of
polarization along the θ1 direction (which will pass through the filter)
and perpendicular to the θ1 direction (which will be absorbed by the
filter). For a wave polarized along the θ2 direction, the amplitude
component along the θ1 direction is given by cos(θ2 − θ1), and the
probability for transmission, given by the wave amplitude squared, is cos2
(θ2 − θ1). That is the prediction of quantum mechanics*

*[...]*


*The key is to allow more than one possibility for the potential result of
a measurement. Orthodox quantum mechanics embraces this notion of multiple
possibilities whenever a quantum state is in a superposition. In the
absence of measurement (and collapse), there is no single definite
potential result. Instead, there are many potential results represented by
many components of the superposition.*

*[...]*


* It is possible to violate Bell’s inequality using either nonlocality or
counterfactual indefiniteness alone, and there are examples of each
approach. To better understand the role of counterfactual indefiniteness,
it is instructive to examine an interpretation of quantum mechanics that
relies solely on counterfactual indefiniteness to violate the inequality.
One of the most popular of these is the “many worlds” interpretation.*

*[...]*


*We claim that the many-worlds interpretation passes the Bell test by
violating counterfactual definiteness, while still respecting locality.
First of all, we argue that after eliminating the nonlocal collapse of
orthodox quantum mechanics, the many-worlds interpretation can be
formulated as a local theory. In particular, the correlated entangled
states used in EPR-Bell experiments can 

Re: Is the "bubble multi-verse" and "qm many-worlds" the same thing?

2018-06-18 Thread Bruce Kellett

From: *Jason Resch* mailto:jasonre...@gmail.com>>


On Mon, Jun 18, 2018 at 7:26 PM, Bruce Kellett 
mailto:bhkell...@optusnet.com.au>> wrote:


From: *Jason Resch* mailto:jasonre...@gmail.com>



On Mon, Jun 18, 2018 at 7:38 AM, Bruce Kellett
mailto:bhkell...@optusnet.com.au>> wrote:

From: *Jason Resch* mailto:jasonre...@gmail.com>>



In the EPR experiment, a pair of photons is created. Each
photon is in a super position of every possible
polarization, and because it is created as a pair, it's dual
in the superposed state always has exactly the opposite
polarization (rotated 180 degrees).



OK.


When you perform a measurement of your left-traveling photon
on Earth, you become entangled (correlated) with it, and all
the possible states of that photon, when measured, leak into
the room, starting with the measuring device, then your
eyes, then your brain, then your notebook, etc. until now
everything is in the room, and soon Earth is now in many
states which contagiously spread from that photon.


OK. Your result (and you) become entangled with your environment.


Also, because the photon you measured was entangled
(correlated) with its pair in the superposition, whatever
result you measure for the photon's polarization tells you
immediately what the polarization of its pair is (in your
branch at least).  So any future communication you get from
me on Pluto will necessarily align with the result you measured.


This is where the mistake creeps in. My measurement tells me
the polarization of the entangled photon in the branch in
which my measurement was made. When you come to measure your
entangled photon on Pluto, how do you know what branch my
measurement was made in? You are at a spacelike separation
from me, and completely independent. So I ask again, how come
you assume that your measurement will be in the same branch
as mine was?


Let's make it more concrete and say there are only 360 possible
polarizations, each having an equal probability.


That is not a very good way to look at it. The photon is not in a
superposition of all possible polarization states. You cannot
write the photon wave function as such a superposition:

 |psi> = Sum_i a_i |i> for i running over all 360
possibilities in the case you outline.

The most you can ever do is write the state as a superposition of
the two possible polarizations in any particular direction. Thus:

   |psi> = (|+> + |->), ignoring normalization factors.

This can be written for |+> and |-> being the polarization
eigenstates in any chosen direction. But not all directions at once.



I see.

Could you explain the point of error in the following paper?  I've 
excerpted the relevant sections if it helps your search.


From: https://arxiv.org/pdf/0902.3827.pdf 



*According to quantum mechanics, whichever measurement is
performed first collapses the entangled twin state superposition
to a single polarization state that is identical for both photons.*
*
*
*[...]*
*
*
*If we wish to know what the probability is of getting the same
measurement for photon 1, we need only figure out what the
probability is for a photon with polarization along θ2 to pass
through a filter oriented along θ1. This probability is easily
calculated according to simple trigonometry. Any arbitrary linear
polarization can be thought of as a superposition of polarization
along the θ1 direction (which will pass through the filter) and
perpendicular to the θ1 direction (which will be absorbed by the
filter). For a wave polarized along the θ2 direction, the
amplitude component along the θ1 direction is given by cos(θ2 −
θ1), and the probability for transmission, given by the wave
amplitude squared, is cos2 (θ2 − θ1). That is the prediction of
quantum mechanics
*
*
*
*[...]*
*
*
*The key is to allow more than one possibility for the potential
result of a measurement. Orthodox quantum mechanics embraces this
notion of multiple possibilities whenever a quantum state is in a
superposition. In the absence of measurement (and collapse), there
is no single definite potential result. Instead, there are many
potential results represented by many components of the superposition.
*
*
*
*[...]*
*
*
*It is possible to violate Bell’s inequality using either
nonlocality or counterfactual indefiniteness alone, and there are
examples of each approach. To better understand the role of
counterfactual indefiniteness, it is instructive to examine an
interpretation of quantum mechanics that relies solely on

Re: Is the "bubble multi-verse" and "qm many-worlds" the same thing?

2018-06-18 Thread Brent Meeker



On 6/18/2018 3:31 AM, Jason Resch wrote:


Block time plus MWI means universes aren't created, they're
all already there.


*Seems like super-determinism to me. You're making a distinction
with no difference. AG
*


Superdeterminism says you and a remote partner could decide to use the 
digits of Pi to pseudorandomly select angles of measurement in a Bell 
experiment, then decide to use the digits of Euler's number. Yet 
somehow, the universe knew you and your friend had this agreement to 
use these digits of these constants,


You keep anthropomorphizing the universe to make super-determinism sound 
ridiculous.  It's nothing more that taking determinism completely 
seriously, no free will by experimenters.  The choice of you and your 
friend was determined by the past.  That's all determinism means.


such that when it generated single pairs of photons, those photons 
would have just the right properties for QM statistics to not be 
violated. Also, the universe knew when you would decide to switch to 
use Euler's number, which perhaps was decided by the closing price of 
the stock market, all this information the universe knew and took into 
account when generating paired photons and embedding a single hidden 
variable with that photon.


Yes, because all those things were determined by what came before them.

This is what Superdeterminism implies.  Superdeterminism is very 
different from regular/plain "determinism", which every physical 
theory is (with the sole exception of wave function collapse).


All other physical theories assume that experimenters can make free 
choices independent of the past history of the world.


Brent





--
You received this message because you are subscribed to the Google Groups 
"Everything List" group.
To unsubscribe from this group and stop receiving emails from it, send an email 
to everything-list+unsubscr...@googlegroups.com.
To post to this group, send email to everything-list@googlegroups.com.
Visit this group at https://groups.google.com/group/everything-list.
For more options, visit https://groups.google.com/d/optout.


Re: Schrodinger's Cat vs Decoherence Theory

2018-06-18 Thread Brent Meeker



On 6/17/2018 10:41 PM, Bruce Kellett wrote:
But the lens doesn't send one color to one photoreceptor and another 
color to a different photorecptor.  It focuses a spot of light on 
several photorecptors and the one with the right pigment fires its 
neuron.  So it is energy detection.


But if you use a different position basis the lens will no longer 
focus point objects to points on the retina.


I don't know enough about the physics of calorimeters as used in HEP 
to comment here. But if temperature changes are measured by bimetals 
or strain gauges, position comes into it in an essential way. 


Most work by measuring a voltage.  But you miss the point.  Those 
position measurements are not essential in the QM sense.  They are 
just changing one classical value into another.  Temperature is the 
first classical level.


Fair enough. I suppose I am just very conscious of the fact that in a 
different position basis all of this physics will be very different. 
The classical universe will not look the same at all.


I guess I don't understand your idea of "position basis".  My 
understanding of linear algebra is that any basis that spans the space 
can be used to represent any relation between structures.  Why should 
choosing a different basis make any difference to the physics aside from 
the simplicity of its representation.  It's just a coordinate basis in 
Hilbert space.  Or are you thinking of bases different from position, 
e.g. momentum, energy, live/dead,...


Brent

--
You received this message because you are subscribed to the Google Groups 
"Everything List" group.
To unsubscribe from this group and stop receiving emails from it, send an email 
to everything-list+unsubscr...@googlegroups.com.
To post to this group, send email to everything-list@googlegroups.com.
Visit this group at https://groups.google.com/group/everything-list.
For more options, visit https://groups.google.com/d/optout.


Re: Is the "bubble multi-verse" and "qm many-worlds" the same thing?

2018-06-18 Thread Brent Meeker



On 6/18/2018 4:27 AM, Jason Resch wrote:



On Sun, Jun 17, 2018 at 9:57 PM, Brent Meeker > wrote:




On 6/17/2018 4:43 AM, Telmo Menezes wrote:

On 17 June 2018 at 13:26,  mailto:agrayson2...@gmail.com>> wrote:


On Sunday, June 17, 2018 at 10:15:05 AM UTC, Jason wrote:



On Sun, Jun 17, 2018 at 12:12 AM, mailto:agrays...@gmail.com>> wrote:



  why do you prefer the MWI compared to the
Transactional Interpretation?
I see both as absurd. so I prefer to assume the wf
is just epistemic, and/or
that we have some holes in the CI which have yet
to be resolved. AG

--



1. It's the simplest theory: "MWI" is just the
Schrodinger equation,
nothing else. (it doesn't say Schrodinger's equation
only applies sometimes,
or only at certain scales)

2. It explains more while assuming less (it explains
the appearance of
collapse, without having to assume it, thus is
preferred by Occam's razor)

3. Like every other successful physical theory, it is
linear, reversible
(time-symmetric), continuous, deterministic and does
not require faster than
light influences nor retrocausalities

4. Unlike single-universe or epistemic
interpretations, "WF is real" with
MWI is the only way we know how to explain the
functioning of quantum
computers (now up to 51 qubits)

5. Unlike copenhagen-type theories, it attributes no
special physical
abilities to observers or measurement devices

6. Most of all, theories of everything that assume a
reality containing
all possible observers and observations lead directly
to laws/postulates of
quantum mechanics (see Russell Standish's Theory of
Nothing, Chapter 7 and
Appendix D).

Given #6, we should revise our view. It is not MWI and
QM that should
convince us of many worlds, but rather the assumption
of many worlds (an
infinite and infinitely varied reality) that gives us,
and explains all the
weirdness of QM. This should overwhelmingly convince
us of MWI-type
everything theories over any single-universe
interpretation of quantum
mechanics, which is not only absurd, but completely
devoid of explanation.
With the assumption of a large reality, QM is made
explainable and
understandable: as a theory of observation within an
infinite reality.

Jason


You forgot #7. It asserts multiple, even infinite copies
of an observer,
replete with memories, are created when an observer does a
simple quantum
experiment. So IMO the alleged "cure" is immensely worse
than the disease,
CI, that is, just plain idiotic. AG

It is important to make the distinction between our intuition and
common sense and actual formal reasoning. The former can guide the
latter very successfully, but the history of science teaches
us that
this is not always the case. You don't provide an argument,
you just
present your gut feeling as if it were the same thing as
irrefutable
fact.


I think Scott Aaronson has the right attitude toward this:

https://www.scottaaronson.com/blog/?p=326



As such a strong believer in quantum computers (he's staked $100,000 
of his own money on the future construction of large scale quantum 
computers), I would love to ask Scott Aaronson what he thinks about 
running a conscious AI on such a quantum computer.  That trivially 
leads to "many worlds" at least as seen by that AI.


If it's so trivial maybe you can explain it.  And you don't have wonder 
about Aaronson thinks, go check his blog.  I'm pretty sure he's posted 
about it.




QM also tells us that Wigner's friend, is no different from that "AI 
running on a quantum computer".


I get kind of tire do being told that QM tells us this or that.  QM is 
just another theory.  Ptolemy's theory told us the Sun went around the 
Earth.  You do realize that QM is inconsistent with GR?


Brent





Jason

--
You received this message because you are subscribed to the Google 
Groups "Everything List" group.
To unsubscribe 

Re: Schrodinger's Cat vs Decoherence Theory

2018-06-18 Thread Bruce Kellett

From: *Brent Meeker* mailto:meeke...@verizon.net>

On 6/17/2018 10:41 PM, Bruce Kellett wrote:
But the lens doesn't send one color to one photoreceptor and another 
color to a different photorecptor.  It focuses a spot of light on 
several photorecptors and the one with the right pigment fires its 
neuron.  So it is energy detection.


But if you use a different position basis the lens will no longer 
focus point objects to points on the retina.


I don't know enough about the physics of calorimeters as used in 
HEP to comment here. But if temperature changes are measured by 
bimetals or strain gauges, position comes into it in an essential way. 


Most work by measuring a voltage.  But you miss the point.  Those 
position measurements are not essential in the QM sense.  They are 
just changing one classical value into another. Temperature is the 
first classical level.


Fair enough. I suppose I am just very conscious of the fact that in a 
different position basis all of this physics will be very different. 
The classical universe will not look the same at all.


I guess I don't understand your idea of "position basis".  My 
understanding of linear algebra is that any basis that spans the space 
can be used to represent any relation between structures.  Why should 
choosing a different basis make any difference to the physics aside 
from the simplicity of its representation.  It's just a coordinate 
basis in Hilbert space.  Or are you thinking of bases different from 
position, e.g. momentum, energy, live/dead,...


Yes, there does seem to be a degree of miscommunication. I am not think 
of different variables such as energy, momentum, or the like. These are 
not different bases, they are different variables and they inhabit 
different Hilbert spaces. So a change of base in one Hilbert space does 
not take you to another space.


No, what I am considering is the possibility of different bases in a 
single space, such as position space. If you assume an eignevector 
interpretation of a set of basis vectors, then a different basis will 
correspond to the eigenvalues of some different operator. It still acts 
in the same, position, space, so it must be regarded as a position 
operator, but it will have quite different physical properties from the 
usual position operator that we use from classical mechanics, where the 
eigenvectors are delta functions along the real line.


Because this will be a different operator, it will correspond to 
different physics. For instance, if the position eigenvalues are 
superpositions of delta functions, corresponding to superpositions of 
different points, the point interactions of particles that we assume in 
constructing the interaction Hamiltonian will be replaced by some set of 
interactions between superpositions of points. This why I suggest that 
the physics will be different. If the physics is the same under this 
basis change, why is there any question about the preferred basis? The 
point is that a change of basis does not mean that we simply go to 
measure some other variable. I think Schlosshauer makes this mistake, if 
I remember correctly; he seems to suggest that the basis choice is 
between position or energy in most cases. That is just wrong.


Bruce











--
You received this message because you are subscribed to the Google Groups 
"Everything List" group.
To unsubscribe from this group and stop receiving emails from it, send an email 
to everything-list+unsubscr...@googlegroups.com.
To post to this group, send email to everything-list@googlegroups.com.
Visit this group at https://groups.google.com/group/everything-list.
For more options, visit https://groups.google.com/d/optout.


Re: Is the "bubble multi-verse" and "qm many-worlds" the same thing?

2018-06-18 Thread Brent Meeker




On 6/18/2018 4:09 AM, Jason Resch wrote:
It will take a lot of work under his approach, but I am not aware of 
any other system proposed by anyone, which even has a chance at this.


Penrose's gravity induced collapse has as good a chance as Bruno's, and 
a better chance of predicting some surprising but true physics. Some 
version of transactional QM also has a chance.  And Omnes' view, summed 
up as, "It's a probabilistic theory, so it predicts probabilities."


Brent

--
You received this message because you are subscribed to the Google Groups 
"Everything List" group.
To unsubscribe from this group and stop receiving emails from it, send an email 
to everything-list+unsubscr...@googlegroups.com.
To post to this group, send email to everything-list@googlegroups.com.
Visit this group at https://groups.google.com/group/everything-list.
For more options, visit https://groups.google.com/d/optout.


Re: Is the "bubble multi-verse" and "qm many-worlds" the same thing?

2018-06-18 Thread Brent Meeker



On 6/18/2018 4:33 AM, Jason Resch wrote:



On Mon, Jun 18, 2018 at 12:01 AM, Brent Meeker > wrote:




On 6/17/2018 2:24 PM, Jason Resch wrote:



On Sunday, June 17, 2018, mailto:agrayson2...@gmail.com>> wrote:



On Sunday, June 17, 2018 at 12:29:35 PM UTC, Jason wrote:



On Sun, Jun 17, 2018 at 6:26 AM,  wrote:



On Sunday, June 17, 2018 at 10:15:05 AM UTC, Jason wrote:



On Sun, Jun 17, 2018 at 12:12 AM,
 wrote:



* why do you prefer the MWI compared to the
Transactional Interpretation? I see both as
absurd. so I prefer to assume the wf is just
epistemic, and/or that we have some holes in
the CI which have yet to be resolved. AG *
-- 




1. It's the simplest theory: "MWI" is just the
Schrodinger equation, nothing else. (it doesn't
say Schrodinger's equation only applies
sometimes, or only at certain scales)

2. It explains more while assuming less (it
explains the appearance of collapse, without
having to assume it, thus is preferred by Occam's
razor)

3. Like every other successful physical theory,
it is linear, reversible (time-symmetric),
continuous, deterministic and does not require
faster than light influences nor retrocausalities

4. Unlike single-universe or epistemic
interpretations, "WF is real" with MWI is the
only way we know how to explain the functioning
of quantum computers (now up to 51 qubits)

5. Unlike copenhagen-type theories, it attributes
no special physical abilities to observers or
measurement devices

6. Most of all, theories of everything that
assume a reality containing all possible
observers and observations lead directly to
laws/postulates of quantum mechanics (see Russell
Standish's Theory of Nothing
,
Chapter 7 and Appendix D).

Given #6, we should revise our view. It is not
MWI and QM that should convince us of many
worlds, but rather the assumption of many worlds
(an infinite and infinitely varied reality) that
gives us, and */explains /*all the weirdness of
QM. This should overwhelmingly convince us of
MWI-type everything theories over any
single-universe interpretation of quantum
mechanics, which is not only absurd, but
completely devoid of explanation. With the
assumption of a large reality, QM is made
explainable and understandable: as a theory of
observation within an infinite reality.

Jason

*
You forgot #7. It asserts multiple, even infinite
copies of an observer, replete with memories, are
created when an observer does a simple quantum
experiment. So IMO the alleged "cure" is immensely
worse than the disease, CI, that is, just plain
idiotic. AG *


There are many atoms, many planets, many solar systems,
many galaxies, many Hubble volumes, and it is believed
many universes.  On what basis are you so certain there
aren't many histories? (That is, other states in the wave
function that are predicted to be there by our well
established scientific theories, but which the theory
explains we cannot see or interact with except in very
limited controlled manners)?
If you find MWI distasteful you might prefer to think of
it as the many-minds interpretation as described by
Heinz-Dieter Zeh, or the "zero-universe interpretation"
as explained by Ron Garrett:
https://www.youtube.com/watch?v=dEaecUuEqfc


I think you are hung up on the "creation", I think it is
conceptually easier to grasp under the understanding that
it is all already there.  If you look at the homepage of
Wei Dai (who founded this e-mailing list
 20 years ago) he
outlines what he 

Re: Mathematics as the result of natural selection

2018-06-18 Thread Brent Meeker




On 6/18/2018 4:44 AM, Steven Ridgway wrote:

On Mon, Jun 18, 2018 at 01:25 Dr Russell Standish wrote:
> "But presumably the argument is about certain cognitive skills which 
helped our species be extraordinarily successful, and also gave us the capability to 
understand algebraic topology."

I've always found it a bit mysterious that humans are so good at abstract 
mathematics. I can see that the evolutionary pressures to improve tool making 
and hunting skills could have given us basic mathematical capabilities - but we 
are far better at it than seems reasonable. i.e. it seems a stretch to imagine 
our ability to understand differential equations and prove Fermat's last 
theorem just fell into place as an accidental by product of something else.

It seems to me that a lot of complex engineering in our brains must exist to 
support the level of abstract reasoning we are capable of - and I don't see 
much evolutionary advantage to explain how this evolved.


It's not that abstract mathematics provides an evolutionary advantage 
(just look at the reproduction rate of mathematicians). But making 
persuasive arguments very much does, and logical inference is important 
in persuasive argument.


Brent

--
You received this message because you are subscribed to the Google Groups 
"Everything List" group.
To unsubscribe from this group and stop receiving emails from it, send an email 
to everything-list+unsubscr...@googlegroups.com.
To post to this group, send email to everything-list@googlegroups.com.
Visit this group at https://groups.google.com/group/everything-list.
For more options, visit https://groups.google.com/d/optout.